SlideShare uma empresa Scribd logo
1 de 26
Manifestations of systemic diseases
1) Bony abnormalities seen in patients with the condition in the figure include all of the following
except:


A. spina bifida

B. pseudoarthrosis

C. lordosis

D. monoarticular arthropathy Correct Choice

E. Kyphosis
This picture demonstrates a café au lait macule and kyphoscoliosis in a patient with
Neurofibromatosis type 1 (NF1). The bony abnormalities seen in these patients include:
kyphoscoliosis, pseudoarthrosis, lordosis, and spina bifida


2) The organism that causes this infection shown in the figure is:

A. C. albicans Correct Choice

B. P. aeruginoas

C. T. tonsuran

D. T. verrucosum

E. C. immitis
The organism that causes erosio interdigitalis blastomycetica is candida albicans.


3) Erythema gyratum repens is known to be associated with all of the following malignancies
except:


A. Bladder carcinoma

B. Cervical carcinoma

C. Breast carcinoma

D. Lung carcinoma

E. Gastric carcinomaCorrect Choice
Erythema gyratum repens presents more commonly in men than in women, and appears clinically
as concentric erythematous rings with trailing scale on the trunk and proximal extremities. The skin
is described as having a "wood grain" appearance. Skin findings often precede the diagnosis of an
associated malignancy. Lung carcinoma is the most commonly associated malignancy, but it has
also been reported in association with breast, cervical, bowel, and bladder cancer.


4) Regarding eruptive xanthomas, which of the following is true?

A. They occur in the setting of familial hyperlipidemia types I, IV, and VCorrect Choice

B. They occur in the setting of familial hyperlipidemia types II and III

C. They are most commonly found on the eyelids

D. They are not related to alcohol consumption

E. Triglyceride levels are usually below 500mg/dl



                                                    1
Eruptive xanthomas generally occur in patients with triglyceride levels of 2000mg/dl or greater.
Associations include poorly-controlled diabetes mellitus, retinoids, estrogens, excessive alcohol
consumption (leading to pancreatitis) and familial hyperlipidemias types I, IV and V. Clinically, they
appear as crops of firm, non-tender yellowish papules with an erythematous border. Most
commonly, they occur on the extensor surfaces, but they can be diffuse. A reduction in triglycerides
and/or tight glucose control usually results in a reduction in the number of lesions


5) Paraneoplastic pemphigus is associated with all of the following underlying malignancies EXCEPT:

A. Non-Hodgkins lymphoma

B. Castleman's tumor

C. Chronic lymphocytic leukemia

D. Thymoma

E. Lung carcinomaCorrect Choice
Paraneoplastic pemphigus has been associated with Non-Hodgkins lymphoma, chronic lymphocytic
leukemia, thymoma, Castleman's tumor, and sarcoma. It has not been associated with lung
carcinoma. Treatment includes management of the underlying malignancy, as well as prednisone or
other immunosuppressive agents


6) What is the most common malignancy associated with this condition at this location?

A. Gastric cancerCorrect Choice

B. Melanoma

C. Breast cancer

D. Thyroid cancer

E. Pancreatic cancer
Malignant acanthosis nigricans usually presents with sudden onset and is rapidly progressive. It may
be associated with diffuse keratodermas of the palms and soles or eruptive seborrheic keratoses


7) The most common location of the lesions in nephrogenic fibrosing dermopathy is:

A. Chest

B. Palms and soles

C. Face

D. Back

E. Lower extremities Correct Choice
The most common location for lesions of nephrogenic fibrosis dermopathy is the lower extremities


8) A 55 year old patient presents with new onset brown macules on arms, legs, face and palms.
She gives a 3 month history of diarrhea, abdominal cramps, weight loss and protein-losing
enteropathy. The most likely diagnosis is:


A. Ulcerative colitis

B. Cowden disease

C. Cronkhite-Canada syndromeCorrect Choice




                                                  2
D. Plummer-Vinson syndrome

E. Peutz-Jeghers syndrome
Cronkhite-Canada is an aquired disease characterized by the development of polyps throughout the
GI tract. Patients can present with hyperpigmented macules as well as the sequella of GI
malabsorption. The malignant transformation of polyps can occur


9) Which of the following is not a feature of Cronkhite-Canada syndrome?

A. Hyperpigmented macules

B. Diarrhea

C. Alopecia

D. Lung carcinomaCorrect Choice

E. Dystrophic nails
Cronkhite-Canada syndrome is a rare, non-familial disease characterized by patchy alopecia, nail
changes or loss, lentigines, inflammatory polyps, abdominal pain, and a protein losing enteropathy


10) A patient with congenital hypertrophy of retinal epithelium is most likely to have:

A. Adenomatous polyposisCorrect Choice

B. Tram-track calcifications on head radiograph

C. Peg-shaped teeth

D. Pheochromocytoma

E. An autosomal dominant mutation in the MSH2 gene
Congenital hyperpigmentation of the retinal pigment (CHRPE) is an early feature of Gardner
syndrome (GS). It is found in approximately 60% of patients with GS. GS is an autosomal dominant
disorder characterized by precancerous intestinal polyposis and subsequent adenocarcinoma of the
gastrointestinal tract. Cutaneous manifestations include epidermoid cysts, osteomas, desmoids and
fibrous tumors. A mutation in the adenomatous polyposis coli (APC) gene, a tumor suppressor gene,
is responsible for the disease. Most patients develop colon carcinoma by the 2nd or 3rd decade.
Therefore, prophylactic colectomy is warranted. Mutations in the MSH2 gene are found in Muir-Torre
syndrome. Pheochromocytomas are found in multiple endocrine neoplasia (MEN) syndromes IIa and
IIb. Tram track calcifications are found in Sturge-Weber syndrome. Peg-shaped teeth are found in
multiple syndromes including ectodermal dysplasia


11) Which of the following may be associated with Graves’ disease?

A. Geographic tongue

B. Madarosis

C. Dermatitis herpetiformisCorrect Choice

D. Hypohidrosis

E. Thick, pale lips
Graves’ disease is a thyrotoxic condition that results from the production of thyroid-stimulating
immunoglobulins (TSI) by stimulated B lymphocytes. The TSI bind to the thyroid-stimulating
hormone (TSH) receptor and mimic TSH thereby stimulating thyroid growth and thyroid hormone
overproduction. Signs and symptoms of Graves’ disease include goiter, tachycardia, exophthalmos,
tremor, sweating, palpitations, smooth moist skin, diarrhea, sleeplessness, irritability, and weight
loss. Autoimmune cutaneous disease may also be associated with Graves’ disease including vitiligo,




                                                  3
dermatitis herpetiformis, herpes gestationis, and pemphigus vulgaris. Cutaneous manifestations of
hypothyroidism include xerosis, hyperhidrosis, yellowish hue, myxedema, and purpura. The hair
may be dry, brittle and coarse; alopecia may be diffuse and/or involve the lateral eyebrow
(madarosis).


12) Which of the following cutaneous findings are characteristic for Vohwinkles Syndrome?

A. Transient erythroderma at birth, palmoplantar keratoderma, follicular hyperkeratosis, scarring
alopecia, dystrophic nails

B. Oral papillomatosis, palmoplantar keratoses, acral keratoses, lipomas, hemangiomas, scrotal
tongue

C. Honeycombed diffuse palmoplantar keratoderma, pseudoainhum with autoamputation, star-
shaped keratosis over knuckles, nail dystrophy, and alopeciaCorrect Choice

D. Rapidly progressive alopecia of all hair-bearing areas, onycholysis, onychoschizia, onychomadesis,
hyperpigmented macules on extremities

E. Dense depigmented lusterless hair, pili torti, doughy skin, diffuse cutaneous hypopigmentation
Vohwinkles Syndrome is an autosomal dominant disorder caused by a mutation of the GJB2 gene
which encodes Connexin 26. Clinical features include deafness, as well as the cutaneous findings
described in choice A


13) In patients with mixed cryoglobulinema associated with hepatitis C, the most likely laboratory
abnormality is:


A. Decreased cryoglobulins

B. Elevated rheumatoid factor Correct Choice

C. + p-ANCA

D. + ANA

E. Elevated hematocrit
In patients with mixed cryoglobulinemia, the most likely laboratory abnormality among the options
listed is an elevated rheumatoid factor


14) cryoglobulinemia are true EXCEPT:

A. Type I is composed of monoclonal IgG and polyclonal IgMCorrect Choice

B. Type II is composed of polyclonal IgG and monoclonal IgM

C. 80% of cases of mixed cryoglobulinemia are associated with Hepatitis C infection

D. Type III is composed of polyclonal IgG and polyclonal IgM

E. None of these answers are correct (all statements are true)
Type I is composed of monoclonal immunoglobulins


15) Which of the following statements regarding porphyrias is TRUE?

A. Delta aminolevulenic acid is the only oxidized porphyrinCorrect Choice

B. Acute intermittent porphyria is the most common form of porphyria

C. Plasma fluoresces at 410 nm in patients with variegate porphyria




                                                  4
D. Griseofulvin is safe for those with variegate porphyria

E. Elevated uroporphyrins are found in the red blood cells of hepatoerythropoietic porphyria
Acute intermittent porphyria (AIP) is the second most common porphyria and is caused by a
deficiency in porphobilinogen (PBG) deaminase, which is located in the cytosol. Patients suffer from
colicky pain, paralysis and psychiatric disorders. There are no specific skin manifestations. PBG and
aminolevulenic acid (ALA) are elevated in the urine. Attacks are precipitated by medications such as
barbiturates, estrogen, griseofulvin, and sulfonamides as well as starvation, fever and infection.
Treatment includes glucose loading and hematin infusion.

Congenital erythropoietic porphyria (CEP) or Günter’s disease is caused by a defect in
uroporphyrinogen III synthase, which is found in the cytosol. Patients are extremely photosensitive
and erythema, blistering and scarring result. Patients present with red urine early in life along with
hypertrichosis and red-stained teeth that fluoresce. Uroporphyrins (URO) are much high than
coproporphyrins (COPRO) in the urine. URO is found in the red blood cells (rbcs) and COPRO is
found in the stool. The rbcs display stable fluorescence.

Porphyria cutanea tarda (PCT) is the most common porphyria and is caused by a deficiency (usually
sporadic) in uroporphyrinogen decarboxylase, which is found in the cytosol. Patients present with
photosensitivity and blistering of sun-exposed areas, especially the dorsal hands. Hypertrichosis and
sclerodermoid changes may occur as well. Liver disease (hepatitis C or alcoholic cirrhosis) is often
present and hemochromatosis may be associated. Urine may fluoresce pink or coral-red with
Wood’s lamp. URO>COPRO in the urine and low levels of COPRO are found in the stool. Treatments
include phlebotomy, antimalarials, and therapy for liver disease if appropriate.

Hereditary coproporphyria (HCP) is caused by a deficiency in coproporphyrinogen oxidase, which is
found in the mitochondria. One-third of patients are photosensitive, and patients suffer
gastrointestinal and neurological symptoms similar to AIP. Urine COPRO is elevated only with
attacks, and COPRO is present in the stool.

Variegate porphyria is the result of decreased activity of protoporphyrinogen oxidase, which is
present in the mitochondria. It combines the skin lesions of PCT with the systemic manifestations of
AIP. Urine COPRO:URO is 1:1 or COPRO> URO to distinguish it from PCT, and PROTO is found in the
stool. The plasma fluoresces at 626nm. Precipitators and treatments are similar to AIP.

Erythropoietic protoporphyria (EPP) is caused by ferrochetalase deficiency, which is present in the
mitochondria. Patients experience immediate burning of the skin with sun exposure. Protoporphyrin
IX, the only oxidized porphyrin in the heme pathway and absorbs in the Soret band (400-410nm).
Patients have erythematous plaques in a photo-distribution. Urine porphyrins are normal. PROTO is
found in the rbcs and the stool. Excessive porphyrins deposited in the liver lead to gallstones and
cirrhosis. Beta carotene may helpful.

Hepatoerythropoietic porphyria (HEP) is essentially a homozygous form of PCT, with deficiency in
uroporphyrinogen decarboxylase. It is clinically similar to CEP with red urine and hypertrichosis,
vesicles and scarring of sun-exposed skin. URO is present in the urine and COPRO in the stool.
PROTO is present in rbcs which distinguishes it from CEP, which was URO in rbcs


16) A young African American patient presents with anemia and spontaneously appearing leg ulcers
over both lateral and medial malleoli. The most likely diagnosis is:


A. Trauma

B. Atherosclerotic disease

C. Sickle cell anemiaCorrect Choice

D. Factitial dermatitis

E. Lupus erythematosus
Sickle cell anemia should be considered most likely in any young African American with spontaneous
leg ulcers. The ulcers are more common in people with severe anemia.




                                                   5
17) The expected histology of a biopsy take from the lesion shown in the image would

A. Schumann bodies

B. Collagen degeneration Correct Choice

C. Granulomatous infiltrate localized to the papillary dermis

D. Increased mucin

E. Elastic fiber degeneration
The histologic findings of necrobiosis lipodica diabeticorum are: dermal granulomatous inflammation
in a horizontal pattern, collagen degeneration, and normal or atrophic epidermis


18) Which of the following is NOT true regarding Cushing's Syndrome?

A. Corticotropin is suppressed with the administration of dexamethasoneCorrect Choice

B. It may be associated with an underlying oat cell lung carcinoma

C. Urine cortisol levels are elevated

D. It may be associated with hypertension and hypokalemia

E. May present with hyperpigmentation and facial plethora
In Cushing's Syndrome, corticotropin is NOT suppressed with the administration of dexamethasone.
The other statements are true


19) Hepatitis C infection is associated with:

A. Gianotti-Crosti syndrome

B. Kapsi’s sarcoma

C. Mixed cryoglobulinemiaCorrect Choice

D. Oral hairy leukoplakia

E. Pityriasis rosea
Cutaneous manifestation associated with hepatitis C include necrolytic acral erythema, porphyria
cutanea tarda, lichen planus, polyarteritis nodosa and mixed cryoglobulinemia. Mixed
cryoglobulinemias is a systemic vasculitis with variable manifestations including palpable purpura,
arthralgias and weakness


20) Which of the following is true regarding piebaldism?

A. It is caused by a mutation in the GJB2 gene

B. It is caused by a mutation in the proto-oncogene c-KITCorrect Choice

C. It is caused by a defect in a protein subunit of a kinase that activates NFkappaB

D. It is caused by defective metabolism of phytanic acid

E. It is caused by a deficiency of fatty aldehyde dehydrogenase
Piebaldism is caused by an autosomal dominant mutation of the proto-oncogene c-KIT which
encodes tyrosine a tyrosine kinase receptor on melanocytes, preventing activation by steel factor




                                                   6
21) An end stage AIDS patient with tuberulosis presents with diffuse hyperpigmentation
of both sun-exposed and unexposed areas. The palmer creases are markedly
hyperpigmented and the patient is very ill. The most likely diagnosis is:

A. Kaposi's sarcoma
B. Argyria
C. Lymphoma
D. Addison diseaseCorrect Choice
E. Tinea versicolor
Addison disease is caused by destruction of the adrenal glands by any cause.
Tuberculosis used to be the primary cause, now the most common cause is auto-immune
destruction. Other causes of adrenal gland destruction include; coccidiomycosis,
cryptococcosis, histoplasmosis, sarcoidosis, metastatic tumor and amyloidosis.

22) A complication seen in women of childbearing age with the condition in the figure is:

A. Pulmonary emboli

B. Lisch nodules

C. Gastrointestinal bleeding

D. Pulmonary lymphangioleiomyoma Correct Choice

E. Atrial septal defect
This picture demonstrates a shagreen patch in a patient with Tuberous sclerosis. A complication
occurring in female patients of child-bearing age is pulmonary lymphangioleiomyoma


23) The histopathologic findings shown in the image is seen in patients with:

A. Cowden's Syndrome Correct Choice

B. Proteus syndrome

C. Bannayan-Riley-Ruvalcaba

D. Rombo syndrome

E. Birt-Hogg-Dube Syndrome
This slide shows lobular formations extending from the epidermis with palisading keratinocytes at
the periphery of the lobules. These features are consistent with trichelemmoma, a characteristic
skin finding of Cowden's disease


24) A representative papule on the dorsal distal finger with a coral bead appearance was biopsied.
The pathology of the lesion demonstrated oncocytic histiocytes with a ground glass appearance, the
diagnosis is:


A. Rheumatoid nodules

B. Granuloma annulare

C. Multicentric reticulohistiocytosis Correct Choice

D. Tendinous xanthomas

E. Erythema elevatum diutinum




                                                       7
This picture and the histologic description are consistent with Multicentric reticulohistiocytosis.
Multicentric reticulohistiocytosis is associated with a symmetric polyarthritis as well as malignancy.
Reported malignancies include melanoma, sarcoma, leukemia, lymphoma, and carcinomas of the
breast, colon, bronchus, cervix, stomach, and ovaries


25) Which of the following is NOT associated with hepatitis C disease?

A. Mixed cryoglobulinemia

B. Single-stranded DNA viridaeCorrect Choice

C. Leukocytoclastic vasculitis

D. Uroporphyrinogen decarboxylase deficiency

E. Single-stranded RNA viridae
Hepatitis C virus (HCV) is a single-stranded RNA virus that is a member of the flaviviridae family.
Approximately 20-30% of patients develop symptoms with acute infection and 70% will progress to
chronic disease. Porphyria cutanea tarda or PCT (uroporphyrinogen decarboxylase deficiency) is
associated with HCV in a substantial percentage of patients; in one study, antibodies to HCV were
found in 82% of PCT patients. Up to 80% of mixed cryoglobulinemia (MC) cases are associated with
HCV. The incidence of lichen planus in HCV patients varies from region to region (0.1-35%). The
histopathology of MC lesions is leukocytoclastic vasculitis. Another relatively common association is
polyarteritis nodosa (PAN) which is also related to hepatitis B infection. Finally, generalized pruritis
is a common complaint of HCV patients


26) All of the following are true regarding incontinentia pigmenti EXCEPT:

A. It is an X-linked recessive disorderCorrect Choice

B. Inflammation and blistering may be followed by hyperkeratotic, verrucous lesions

C. It is caused by a mutation in the NEMO gene

D. It is associated with cerebellar ataxia

E. It is associated with coloboma and retinal detachment
Incontinentia pigmenti is an X-linked dominant disorder caused by a mutation in the NEMO gene.
Females only present at birth with linear lesions of inflammation and blistering (stage 1), followed
by hyperkeratotic verrucous areas (stage 2), hyperpigmentation (stage 3), and then atrophy (stage
4). Systemic findings include psychomotor retardation, microcephaly, seizures, cerebellar ataxia,
coloboma, and retinal detachment


27) Which of the following is a paraneoplastic disease most often associated with lung carcinoma?

A. Acanthosis nigricans

B. Dermatomyositis

C. Paraneoplastic pemphigus

D. Erythroderma

E. Hypertrichosis lanuginosa acquisitaCorrect Choice
Hypertrichosis lanuginosa acquisita is the abrupt onset of downy, soft, non-pigmented hair of the
face, trunk, and extremities. It may have an associated glossitis. It is associated with underlying
lung carcinoma and may resolve with treatment of the underlying malignancy


28) Patients with this syndrome, as well as their family members, should undergo periodic
surveillance for renal cell carcinoma:



                                                   8
A. Pseudoxanthoma Elasticum

B. Fabry's Disease

C. Birt-Hogg-Dube SyndromeCorrect Choice

D. Osler-Weber-Rendu

E. Nail-Patella Syndrome
Strong evidence suggests that BHDS and bilateral renal tumors co-segregate in an autosomal
dominant fashion. Histologically, these renal tumors are oncocytomas, chromophobe adenomas, and
papillary renal cell carcinoma. Periodic surveillance of patients and their relatives for renal cell
carcinoma is recommended


29) You are called to evaluate a patient on the oncology service with the lesions pictured. Which of
the following statements is NOT true regarding this disease process? (Photo courtesy of
Derm101.com)


A. This patient will most likely die from complications of the underlying malignancy.

B. Skin biopsy would reveal suprabasal acantholysis and dyskeratosis.

C. This patient most likely has an underlying gastric carcinoma.Correct Choice

D. Direct immunofluorescence of perilesional skin would demonstrate intercellular IgG and granular
C3 at the dermoepidermal junction.

E. Skin biopsy would reveal vacuolar interface dermatitis with a lichenoid infiltrate.
Paraneoplastic pemphigus presents with painful oral erosions/ stomatitis as well as a polymorphous
skin eruption. Most patients die from complications of the underlying malignancy. The condition is
associated with Non-Hodgkins lymphoma, chronic lymphocytic leukemia, thymoma, Castleman's
tumor, and sarcoma. Statements A, C, D, and E are correct


30) All of the following are true regarding the condition pictured EXCEPT:

A. It is associated with HLA-DQ2, HLA-DR3, and HLA-B8

B. Only 20% of patients have a gluten-sensitive enteropathyCorrect Choice

C. Perilesional direct immunofluroescence shows granular IgA in the dermal papillae and at the
dermoepidermal junction

D. Cutaneous findings are due to autoantibodies to epidermal transglutaminase

E. It is associated with Hashimoto's thyroiditis
All of the statements regarding dermatitis herpetiforms, or "Duhring's Disease," are true except for
statement B. Virtually all DH patients have gluten-sensitive enteropathy, although only 20% of
them have symptoms (such as diarrhea, steatorrhea, weight loss, bloating, and malabsorption).


31) Which of the following is true regarding cutaneous associations with hepatitis C virus (HCV)
infection?


A. Pruritus in the setting of chronic HCV infection is generally correlated with elevated bile salt levels
in the setting of liver failure

B. Cutaneous reactions to HCV treatment are less common with interferon/ribavirin combination
treatment than with treatment with interferon alone

C. HCV-related porphyria cutanea tarda may be caused by decompartmentalization of iron stores
and resultant oxidation of uroporphyrinogen decarboxylaseCorrect Choice




                                                    9
D. Polyarteritis nodosa is associated with HCV infection but not hepatitis B virus infection

E. Erosive mucosal lichen planus has a weaker association with HCV than does cutaneous lichen
planus
Erosive mucosal variant of lichen planus has the strongest association with HCV. The pathogenesis
of pruritus in the setting of chronic HCV infection may be related to elevated bile salt levels in the
setting of liver failure, but there is not always a direct correlation between serum bile salt level and
degree of pruritus. Polyarteritis nodosa is associated with both HCV infection and hepatitis B virus
infection. Statement D is correct. Cutaneous reactions to HCV treatment are more common with
interferon/ribavirin combination treatment than with treatment with interferon alone.


32) Which of the following is TRUE about diabetic-related skin disease?

A. Necrobiosis lipoidica affects approximately 20% of diabetics

B. Bullae are common on the thighs

C. Diabetic dermopathy affects the upper back

D. Yellow skin affects the majority of diabetic patients

E. Direct immunofluorenscence is usually negative in bullous diabeticorumCorrect Choice
Approximately 30% of diabetic patients have cutaneous manifestations of their disease. Diabetic
dermopathy, or “shin spots” is the most common cutaneous association with diabetes. Patients are
generally long-standing diabetics, and are associated with symmetrical brownish, atrophic plaques
on the shins. Acanthosis nigricans is more common in black and Hispanic diabetics, and may be a
component of the HAIRAN (hyperandrogen, insulin resistance, acanthosis nigricans) syndrome.
Hyperpigmented velvety plaques are most often found in the flexures. Waxy, thick skin and limited
joint mobility are both related to poor glucose control. Scleredema diabeticorum, which most often
causes thickened plaques of the upper back and neck most often affects type II diabetics.
Necrobiosis lipoidica diabeticorum (NLD) is present in only 0.3 to 3% of diabetics, but perhaps 20%
of patients with NLD have diabetes or glucose intolerance. Sharply-demarcated yellow-brown,
telangiectatic plaques are found on the tibial surfaces. Ulceration may occur. Yellow skin may affect
approximately 10% of diabetics and presents diffusely. Serum carotene levels may be elevated.
Perforating disorders may be related, especially in the setting of renal disease. In the setting of
hypertriglyceridemia, eruptive xanthomata may be present. The association between diabetes and
granuloma annulare (GA) is controversial. If truly an association, the generalized and perforating
forms of GA are most commonly associated. Infectious associations include candidiasis, which may
present an angular cheilitis, chronic paronychia, interdigital or intertriginous disease. Cutaneous
bacterial infections, including group A and B streptococci, pseudomonas aerugenosa and
corynebacterium are more common. Dermatophyte infections are not more common, although tinea
pedis may increase the risk for cellulitis in diabetic patients. Finally, rhinocerebral mucormycosis is a
potential disasterous complication of uncontrolled hyperglycemia with ketoacidosis. Mortality rates
can approach 35%.


33) The most likely underlying malignancy associated with the lesions shown is:

A. Breast Cancer

B. Waldenstrom's macroglobulinemia

C. Acute myelogenous leukemia Correct Choice

D. Castleman's tumor

E. Multiple myeloma
These are typical lesions of Acute Febrile Neutrophilic dermatosis (Sweet's Syndrome), a
paraneoplastic disease most strongly associated with Acute myelogenous leukemia


34) The most common autoimmune disease associated with hepatitis C is:


                                                   10
A. Myasthenia gravis

B. Autoimmune thyroiditis Correct Choice

C. Aplastic anemia

D. Rheumatoid arthritis

E. Lymphocytic sialadenitis
The most common autoimmune disease seen in association with Hepatitis C is autoimmune
thyroiditis.


35) This patient with the above findings also has recurrent bouts of rectal bleeding and cramping.
The most likely underlying disease is:


A. Henoch-schonlein purpura

B. Gardner’s syndrome

C. Polyarteritis nodosum

D. Ulcerative colitis Correct Choice

E. Dermatitis herpetiformis
These cobblestone mucosal lesions are pyostomatitis vegetans. They are seen in those with
ulcerative colitis


36) The shoulder pad sign has been described in which disease?

A. cushings disease

B. nephrogenic fibosing dermopathy

C. dermatomyositis

D. systemic lupus erythematosis

E. systemic amyloidosisCorrect Choice
The shoulder pad sign has been described in the setting of late systemic amyloidosis and is due to
the direct deposition of amyloid in the deltoid muscles


37) Hypothyroidism may result in all of the following cutaneous findings except:

A. Purpura

B. Yellowish hue

C. All of these answers are correctCorrect Choice

D. Increase in the percentage of telogen hairs

E. Madarosis
All of the above are non-specific cutaneous manifestations of hypothyroidism: an increase in
percentage of telogen hairs, yellowish hue caused by carotenemia, purpura secondary to impaired
wound healing, and loss of the lateral third of the eyeborw (madarosis)


38) The most likely underlying diagnosis of the patient in the figure is:

A. SLE




                                                    11
B. Endocarditis

C. Mycosis Fungoides

D. Multicentric Reticulohistiocytosis

E. Dermatomyositis Correct Choice
This patient has periungal telangiectasias. It is commonly seen in those with dermatomyositis


39) Which of the following diseases is caused by an enzymatic defect that occurs in the
mitochondria?


A. Erythropoeitic ProtoporphyriaCorrect Choice

B. Hepatoerythropoetic Porphyria

C. Congenital Erythropoeitic Porphyria

D. Acute Intermittent Porphyria

E. Porphyria Cutanea Tarda
Defects in many of the enzymes involved in heme synthesis are responsible for porphyrias. The first
and last three steps of the heme synthesis pathway occur in the mitochondria


40) Which of the following is (are) characteristic of basal cell nevus syndrome?

A. Tram-track calcifications

B. Telangiectasias

C. Autosomal recessive inheritance

D. NEMO gene mutation

E. ColobomasCorrect Choice
Basal cell nevus syndrome (Gorlin syndrome) is caused by an autosomal dominant mutation in
PTCH gene that encodes PTC protein involved in sonic hedge hog pathway. This mutation leads to
loss of inhibition of smoothened (SMO) leading to an increased expression of other genes.
Cutaneous findings include nevoid basal cell carcinomas, milial cysts, epidermoid cysts and palmar-
plantar pits. Neurological findings include calcifications of the falx cerebri, agenesis of the corpus
callosum, mental retardation, medulloblastoma and spina bifida. Ocular findings include blindness,
cataracts, colobomas and strabismus. Other findings include odontogenic cysts of the jaw, frontal
bossing, bifid ribs, and pectus deformity. NEMO gene mutations are found in incontinentia pigmenti.
Tram-track calfications are seen Sturge-Weber syndrome. Telangiectasias are seen in ataxia-
telangiectasia among others


41) All of the following statements are true regarding this condition EXCEPT:

A. 0.3-3% of diabetics have this skin condition

B. There is no impact of tight glucose control on the likelihood of developing this condition

C. This condition is associated with increased dermal mucinCorrect Choice

D. Diabetes or glucose intolerance is found in 20% of these patients

E. This condition may be associated with cutaneous anesthesia, hypohidrosis, and partial alopecia
Necrobiosis Lipoidica Diabeticorum (NLD) is found in 0.3-3% of diabetics. Approximately 20% of
NLD patients have diabetes or glucose intolerance. It presents with single or multiple red-brown
papules which progress to sharply demarcated yellow-brown atrophic, telangiectatic plaques with
violaceous, irregular borders; common sites include the shins. Cutaneous anesthesia, hypohidrosis,



                                                  12
and partial alopecia can be found. Pathology shows palisading granulomas containing degenerating
collagen (necrobiosis); with NO increase in dermal mucin. There is no impact of tight glucose control
on the likelihood of developing NLD


42) A patient with esophageal carcinoma has scale plaques on the ears, hands and feet. He may
have:


A. Dermatomyositis

B. Sweet’s syndrome

C. Bazex’s syndromeCorrect Choice

D. Paraneoplastic pemphigus

E. Multicentric reticulohistiocytosis
Bazex’s syndrome (acrokeratosis paraneoplatica of Bazex) is characterized by psoriasiform changes
of the hands, ears, feet, and nose. Involvement of the nails may be indistinguishable from psoriasis.
It occurs in association with malignancies of the upper aerodigestive tract


43) Which of the following is an autosomal recessive disorder characterized by pili torti and
deafness?


A. Waardenburg Syndrome

B. Bjornstad's SyndromeCorrect Choice

C. Refsum Disease

D. Sjorgen-Larsson Syndrome

E. Cockayne Syndrome
Waardenburg Syndrome is an autosomal dominant disorder. Sjorgen-Larsson Syndrome is an
autosomal recessive disorder characterized by a triad of ichthyosis, di- or tetraplegia, mental
retardation. Refsum Disease is an autosomal recessive disorder whose features include deafness,
mild ichthyosis in adulthood, and accentuated palmar creases. Bjornstad's Syndrome is an
autosomal recessive disorder characterized by pili torti and deafness. Cockayne Syndrome is an
autosomal recessive disorder whose features include photosensitivity, bird-headed facies, and
"Mickey-Mouse" ears, as well as deafness


44) Triple palms are a cutaneous manifestation associated with which of the following
malignancies?


A. Renal carcinoma

B. Colon carcinoma

C. Pancreatic carcinoma

D. Prostate carcinoma

E. Lung carcinomaCorrect Choice
Tripe palms are rugose thickening of the palms which are nearly always associated with internal
malignancy. When tripe palms are present without other cutaneous findings, the most likely internal
malignancy is lung cancer. Tripe palms in association with acanthosis nigricans is most likely to be
associated with gastric cancer


45) Mutations in the STK11 gene encoding a serine threonine kinase are seen in:




                                                 13
A. Muir-Torre Syndrome

B. Bannayan-Riley-Ruvalcaba Syndrome

C. Cronkhite-Canada Syndrome

D. Peutz-Jeghers SyndromeCorrect Choice

E. Birt-Hogg-Dubbe Syndrome
Peutz-Jeghers Syndrome is an autosomal dominant syndrome. 50% of patients have mutations in
the STK11 gene which encodes a serine threonine kinase. It presents with periorificial and mucosal
lentigines beginning in infancy, as well as hamartomatous intestinal polyps with intussusception and
bleeding; gallbladder, pancreatic, breast, ovarian, and testicular cancer


46) When metastases to the skin occur from a thyroid malignancy, they are usually due to:

A. Medullary carcinoma

B. Papillary adenocarcinomaCorrect Choice

C. Cutaneous metastases have not been reported in association with thyroid malignancies

D. Anaplastic carcinoma

E. Follicular carcinoma
Metastases to the skin from a thyroid malignancy are rare, but most reported cases occur with
papillary adenocarcinoma


47) An uncommon complication of treatment with potassium iodide is:

A. Acute generalized exanthematous pustulosis

B. Exacerbation of lichen planus

C. Erythema nodosum

D. Wolff-Chiakoff effect Correct Choice

E. Hyperhidrosis
An uncommon complication of treatment with potassium iodide is the Wolff-Chiakoff effect


48) A patient presents with diffuse waxy keratoderma of the palms and soles as well as oral hairy
leukoplakia. Which of the following statements regarding this condition is NOT true?


A. Features may include squamous cell carcinomas arising from keratodermic skin

B. None of these answers are correct (all are true)

C. Family members should be advised to undergo cancer screening

D. This condition is autosomal recessiveCorrect Choice

E. This condition may be associated with esophageal carcinoma
Howell-Evans syndrome is an autosomal dominant disorder that presents with diffuse waxy
keratoderm of the palms and soles, as well as oral hairy leukoplakia and squamous cell carcinoma
arising from keratodermic skin. It is associated with esophageal carcinoma; cancer screening in
family members is advised


49) The presence of antibodies to c-ANCA is characteristically seen in patients with which disease?



                                                 14
A. Nodular vasculitis

B. Polyarteritis nodosa

C. Churg-Strauss disease

D. Ulverative colitis

E. Wegener’s granulornatosis Correct Choice
Wegener’s granulomatosis is a necrotizing granulomatous disorder that most commonly affects the
upper and lower respiratory tracts, kidneys, and eye. Cytoplasmic pattern antineutrophil
cytoplasmic autoantibody (anti -proteinase-3) is often positive in this disease. Churg-Strauss is
associated with a positive p-ANCA


50) Patients with the condition shown who also have JXG are at increased risk for developing:

A. Juvenile chronic myelogenous leukemia Correct Choice

B. Non-Hodgkin's Lymphoma

C. Medullary carcinoma of the thyroid gland

D. Breast cancer

E. Esophageal cancer
The picture demonstrates a plexiform neuroma in a patient with NF I. Patient with NF 1 who have
multiple juvenile xanthogranulomas are at 20 times greater risk for developing juvenile chronic
myelogenous leukemia


51) Hyperkeratotic follicular nasal papules have been described as a paraneoplastic sign in the
setting of which neoplasm?


A. Castleman's tumor

B. Renal cell carcinoma

C. AML

D. Adenocarcinoma of the lung

E. Multiple myelomaCorrect Choice
Hyperkeratotic follicular nasal papules have been described as a paraneoplastic phenomenon in the
setting of multiple myeloma. AML is associated with Sweet's syndrome and Castleman's tumor is
associated with paraneoplastic pemphigus


52) A patient with gluten-sensitive enteropathy presents with vesicles on the extensor surfaces of
the extremities. What findings are most likely on a perilesional biopsy?


A. Granular IgA at the dermoepidermal junction on direct immunofluorescenceCorrect Choice

B. Linear C3 and IgG at the dermoepidermal junction on direct immunofluorescence

C. Leukocytoclastic vasculitis

D. Linear IgA surrounding vessels on direct immunofluorescence

E. Pautrier’s micro-abscesses in the epidermis on H & E
This patient has dermatitis herpetiformis (DH). DH is characterized by itchy papulovesicles on
extensor surfaces of the extremities. Neutrophilic infiltrates at the dermal papillae with vesicle
formation are found on histopathologic examination. On immunofluorescence, granular IgA
(directed against transglutaminase) deposits are found in perilesional skin. Over 90% of patients




                                                 15
have gluten-sensitive enteropathy of varying severity. Dapsone is almost universally therapeutic for
the skin disease, but not the enteropathy. A gluten free diet, although difficult to maintain, treats
the enteropathy. The majority of DH patients have the HLA class II DQ2 genotype. On indirect
immunofluorescence, one may find antigliadin, antiendomyseal or antireticulin antibodies.
Autoimmune thyroid disease (especially Hashimoto’s thyroiditis), enteropathy-associated T-cell non-
Hodgkin’s lymphoma and insulin-dependent diabetes are the most common autoimmune
associations. Pautrier’s micro-abscesses are found in mycosis fungoides. Leukocytoclastic vasculitis
is not a feature of DH


53) Patients with Werner’s syndrome typically experience which of the following types of cardiac
disease?


A. Premature atherosclerosisCorrect Choice

B. Mitral valve prolapse

C. Hypertrophic cardiomyopathy

D. Aortic aneurysms

E. Cardiomegaly
Werner’s syndrome or Progeria is caused by autosomal recessive mutations in WRN (Recql2) gene,
which encodes DNA helicase. This defect leads to defects in DNA repair and replication. Patients
prematurely age and essentially experience many diseases of aging early in childhood and teen
years. They exhibit tight atrophic skin, relatively large heads for body size, leg ulcers and cannities.
Patients experience early, accelerated atherosclerosis leading to death by myocardial infarction. In
addition, type II diabetes, cataracts, osteoarthritis, osteoporosis and hypogonadism are features


54) A patient presents with a rapid increase in the number and size of these lesions. Your next step
should be to:


A. Reassure the patient that the lesions are benign

B. Order a chest xray

C. Refer the patient for a workup to rule out a gastrointestinal malignancy.Correct Choice

D. Discuss with the patient's PCP re/ altering their diabetic regimen

E. Order a CBC and SPEP/UPEP
The sign of Lesser-Trelat is a rapid increase in the size and/or number of seborrheic keratoses. It
can be seen in association with acanthosis nigricans and tripe palms, and may have an associated
generalized pruritus. It is associated with gastric or colon carcinoma, can appear before or after the
malignancy, and may improve with treatment of the underlying malignancy


55) A 6 month-old has a verrucous plaque on the mucosal surface of the lower lip. Skin biopsy is
consistent with Riga-Fede disease. You should refer the patient to:


A. A hematologist

B. An otolarngologist

C. A gastroenterologist

D. An ophthalmologist

E. A neurologistCorrect Choice
Riga-Fede disease is a benign ulcerative granulomatous process that occurs in reaction to chronic,
repetitive trauma of the oral mucosa by the teeth. Clinically, it appears as firm, verrucous plaques.
It may be associated with an underlying developmental anomaly or underlying neurologic disorder




                                                   16
56) The clinical findings in figure 3 are associated with which disorder?

A. Dermatomyositis

B. Graves disease Correct Choice

C. Rheumatoid Arthritis

D. Myasthenia gravis

E. Sarcoidosis
This slide shows pretibial myxedema in a patient with Grave’s disease


57) All of the following are true regarding Cockayne Syndrome EXCEPT:

A. None of these answers are correct Correct Choice

B. It is caused by an inability to repair cyclobutane dimers

C. Inheritance is autosomal recessive

D. It is associated with retinal pigment degeneration with a "salt and pepper" appearance

E. It is associated with basal ganglia calcifications
Cockayne Syndrome is an autosomal recessive disorder caused by a mutation of an unknown gene
that results in the inability to repair cyclobutane dimers induced by UV exposure. Cutaneous
findings include photosensitivity, "bird-headed" facies, and "Mickey Mouse" ears. Statements A-D
are true regarding the syndrome


58) Squamous cell carcinoma is seen in which syndrome?

A. Nicolau-Balus syndrome

B. Bazex syndrome Correct Choice

C. Gorlin syndrome

D. Rombo syndrome

E. Rasmussen syndrome
Basex syndrome, also paraneoplastic acrokeratosis, presents with symmetric erythematous, nearly
violaceous, psoriasiform dermatoses of the hands, feet, ears and nose. The syndrome is nearly
associated with an underlying malignancy, usually squamous cell carcinoma of the upper
aerodigestive tract


59) Which of the following is true regarding treatment of inflammatory dermatoses with potassium
iodide?


A. The Wolff-Chaikoff effect must be considered

B. The Wolff-Chaikoff effect must be considered and Binding of excess organic iodide in the thryoid
gland may occur

C. Thyroid hormone synthesis may be inhibited

D. All of these answers are correctCorrect Choice

E. Binding of excess organic iodide in the thryoid gland may occur




                                                    17
The Wolff-Chaikoff effect is described as the binding of excess organic iodide in the thyroid gland
with resultant inhibition of thyroid hormone synthesis. This can occur in the setting of patients with
erythema nodosum (or other inflammatory dermatoses) being treated with potassium iodide


60) Which of the following statements regarding multiple endocrine neoplasia syndromes is true?

A. MEN Type IIA is also known as Wermer's Syndrome

B. Multiple mucosal neuromas are seen in association with MEN Type IIA

C. Lichen or macular amyloidosis is seen in association with MEN Type IIACorrect Choice

D. Patients with MEN Type IIB are at increased risk for developing follicular thyroid carcinoma

E. MEN Type IIB is also known as Sipple's Syndrome
MEN Type I is also known as Wermer's Syndrome. MEN Type IIA is also known as Sipple's
Syndrome. Mucosal neuromas are seen in association with MEN Type IIB. Statement D is correct.
Patients with MEN Type IIB are at increased risk for developing medullary thyroid carcinoma


61) All of the following are true regarding the condition pictured EXCEPT:

A. Men with this condition may be at increased risk for lymphoma

B. Associated calcinosis cutis may be seen in pediatric patients

C. Age-appropriate cancer screening is recommended

D. May be associated with a psoriasiform scalp dermatitis

E. Elevated aldolase is more specific than CPKCorrect Choice
All of the statements regarding dermatomyositis are true except for statement E. Elevated muscle
enzymes are found in dermatomyositis, but elevated CPK is more specific than aldolase. Diagnosis is
also established with muscle biopsy showing evidence of inflammation and abnormal EMG


62) This autosomal dominant condition is characterized by trichodiscomas, fibroepithelial polyps,
and fibrofolliculomas:


A. POEMS syndrome

B. Cowden's disease

C. Rombo syndrome

D. Proteus syndrome

E. Birt-Hogg-Dube syndrome Correct Choice
Birt-Hogg-Dube is an autosomal dominant condition characterized by trichodiscomas, fibroepilethial
polyps and fibrofolliculomas


63) A patient on hemodialysis presents with indurated plaques having a peau d’orange texture on
the bilateral lower legs. Which of the following statements is TRUE?


A. The face is usually is affected

B. Dapsone will likely be effective treatment

C. The diagnostic histopathological findings include acanthosis and hyperkeratosis

D. The palms and soles are usually affected




                                                  18
E. Serum protein electrophoresis should be performedCorrect Choice
This patient likely has nephrogenic fibrosing dermopathy (NFD). NFD is an acquired, idiopathic
disorder that occurs in renal disease patients. It resembles scleroderma or eosinophilic fasciitis
clinically and scleromyxedema histopathologically. Large areas of indurated skin with fibrotic
nodules and plaques develop. The extremities are most commonly involved, followed by the trunk.
The face, palms and soles are almost never involved. Histopathologically, NFD displays a
proliferation of dermal fibroblasts and dendritic cells, thickened collagen bundles, increased elastic
fibers, and mucin deposition. Serum protein electrophoresis and immunoelectrophoresis results are
negative unlike scleromyxedema, and may be helpful to distinguish the two diseases. NFD is usually
a chronic, progressive condition, and favorable responses to medical intervention are anecdotal


64) All of the following are true regarding calciphylaxis except:

A. Seen in end-stage renal disease

B. Vascular mural calcification occurs late in the processCorrect Choice

C. Association with an elevated calcium: phosphate product

D. Association with hyperparathyroidism

E. High morality rate is due to sepsis
Calciphylaxis is a rare, life threatening disorder that is associated with end stage renal disease.
Clinically, patients develop a livedoid purpura and enlarging, tender, indurated subcutaneous plaque
typically on the legs or abdomen. These plaques are subject to ischemia, skin necrosis, and
secondary infection, sepsis, and death. Histologic examination of calciphylaxis is characterized by
the triad of small vessel mural calcivication, extravascular calcification, and vascular thrombosis. It
appears that vascular mural calcification is an early and essential process in the development of
calciphylaxis


65) All of the following are seen more commonly in Crohn's disease than in ulcerative colitis
EXCEPT:


A. Perineal fistulas

B. Pyostomatitis vegetansCorrect Choice

C. Oral cobblestoning

D. Perineal fissures

E. Polyarteritis nodosa
Pyoderma vegetans consists of vegetating plaques and vesicopustules of intertriginous areas than
heal with hyperpigmentation. When the process involves mucosal surfaces it is called pyostomatitis
vegetans. These processes are associated with ulcerative colitis, not Crohn's disease


66) Which of the following is FALSE regarding Fabry’s disease?

A. It is associated with acral parasthesias

B. It is inherited in an X-linked recessive fashion

C. The etiology is a defect in alpha-galactosidase A

D. It may be associated with renal failure

E. It is inherited in an X-linked dominant fashionCorrect Choice
Fabry’s disease is an X-linked lysosomal disorder that leads to excessive deposition of neutral
glycosphingolipids in the vascular endothelium. The disorder is caused by a deficiency of alpha-
galactosidase-A leading to progressive endothelial accumulation of glycosphingolipids. This



                                                      19
accumulation accounts for the associated clinical abnormalities of skin, eye, kidney, heart, brain,
and peripheral nervous system. Acroparesthesias are a frequent presenting symptom of Fabry’s
disease. The primary cutaneous manifestation are angiokeratomata (angiokeratoma corporis
diffusum), which are most common in a “bathing-trunk” distribution. Lens opacities and retinal and
conjunctival vascular malformations may be found in the eyes. Patients may experience cardiac
disease, stroke or renal failure


67) Which of the following statements about multiple endocrine neoplasia (MEN) syndromes is
FALSE?


A. MEN I is associated with angiofibromas and collagenomas

B. MEN IIa is related to a defect in RET proto-oncogene

C. MEN I, IIa and IIb are all inherited in an autosomal dominant fashion

D. MEN IIa is associated with multiple mucosal neuromasCorrect Choice

E. MEN IIa and IIb are both associated with medullary thyroid carcinoma
Multiple endocrine neoplasia (MEN) syndromes are divided into types I, IIa and IIb. All are
autosomal dominantly inherited. Type I is associated with mutations in MENI; type IIa and IIb
involve mutations of RET, which encodes a tyrosine kinase receptor. Type I has the following
cutaneous features: facial angiofibromas, collagenomas, lipomas, hypopigmented macules and café-
au-lait macules. Type IIa displays macular amyloidosis, while IIb features multiple mucosal
neuromas. Systemic features of type I include peptic ulcer disease (as part of Zollinger-Ellison
syndrome), parathyroid hyperplasia or adenoma, pancreatic tumors and pituitary neoplasia. Type
IIa is characterized by Zollinger-Ellison syndrome, medullary thyroid carcinoma,
pheochromocytoma, and parathyroid neoplasia. Type IIb displays marfanoid habitus, Hirschsprung
disease, medulloblastoma, medullary thyroid carcinoma, pheochromocytoma and ocular neuromas


68) Which of the following is true regarding nephrogenic fibrosing dermopathy?

A. Is associated with a dramatic increase in dermal mucin

B. May be associated with antiphospholipid antibodiesCorrect Choice

C. Has a rapidly progressive but reversible course

D. Is associated with a paraproteinemia

E. Is associated with peripheral eosinophilia
Nephrogenic fibrosing dermopathy has an indolent course and treatment is usually not satisfactory.
It is not associated with a paraproteinemia or peripheral eosinophilia. Some patients have been
reported to have antiphospholipid antibodies. Histopathology demonstrates a minimal to slight
increase in dermal mucin


69) All of the following are seen more commonly in ulcerative colitis than in Crohn's disease except:

A. Pyoderma gangrenosum

B. Erythema nodosum

C. Chronic apthous ulcers

D. Pyoderma vegetans

E. Polyarteritis nodosaCorrect Choice
Polyarteritis nodosa is associated with Crohn's disease and not ulcerative colitis




                                                  20
70) A patient with multiple red-brown papules arranged around the nail folds and “operaglass
fingers” may have all of the following associated findings except:


A. Underlying malignancy

B. Fever and weight loss

C. Multinucleate giant cell infiltrate

D. Progressive cutaneous diseaseCorrect Choice

E. Mutilating arthritis
Multicentric reticulohistiocytosis is a rare disorder characterized by mutilating arthritis and
cutaneous nodules. Involvement may also include the mucosa, synovia, and internal organs. On the
skin, patients may have non-tender, reddish-brown nodules on the face, hands (coral bead
appearance), ears, forearms, scalp, neck, and eyelids. These nodules have a histiocytic infiltrate
with multinucleated giant cells with a ground glass appearance. Multicentric reticulohistiocytosis has
been reported in association with carcinoma of the colon, breast bonchus, cervix, ovary, stomach,
mesothelioma, and melanoma


71) Each of the following demonstrates a vasculitis except:

A. Henoch-Schoenlein purpura

B. Granuloma faciale

C. Type 1 cryoglobulinemiaCorrect Choice

D. Leukocytoclastic vasculitis

E. Wegener’s granulomatosis
In type I cryoglobulinemia, monoclonal IgG or IgM cryoglobulins are found often in association with
lymphoma, leukemia, Waldenstrom’s macroglobulinemia, or multiple myeloma. On histopathology,
type I cryoglobulinemia is characterized by the deposition of precipitated amorphous cryoglobulins
on the endothelium and throughout the vessel wall. The precipitates stain with PAS stain. An
inflammatory infiltrate is typically lacking in contrast to mixed cryoglobulinemia (which shows a
leukocytoclastic vascultitis)


72) Regarding carcinoid syndrome, which of the following is TRUE?

A. The most common location for the tumor is the rectum

B. Somatostatin is a preferred treatmentCorrect Choice

C. VMA is elevated in the urine

D. Symptoms are caused by metastases to the skin

E. The tumors should not be removed
Carcinoid syndrome is caused by a tumor originating in the endocrine argentaffin cells. In order of
descending frequency, the most common locations are appendix > small bowel > rectum. Patients
experience facial flushing that descends anatomically, diarrhea, hyperhidrosis and bronchial
reactivity. A pellagra-like eruption may occur as well as telangiectasia and sclerodermoid changes.
Patients experience symptoms once the disease metastasizes to the liver or if it did not involve the
GI tract initially. Diagnosis can be made finding elevated 5-hydroxyindolacetic acid (5-HIAA) in the
urine. Nitrosonaphthol turns urine purple if 5-HIAA levels are significantly elevated, and can be used
as a screening test. Bananas, tomatoes, plums, avocadoes and eggplant can increase levels of 5-
HIAA. Treatment involves removal of the tumor along with medical therapy if appropriate. Agents
used include somatostatin, methylsergide, cyproheptadine, and beta blockers. Vanillylmandelic acid
(VMA) is an important urinary metabolic product of epinephrine and norepinephrine and is detected
in the diagnosis of pheochromocytoma



                                                 21
73) Which of the following statements regarding Menkes Disease is true?

A. Diagnosis is made by elevated serum ceruloplasmin

B. Males with the disease may have cutaneous hypopigmentation

C. It is caused by a mutation of the ATP7A geneCorrect Choice

D. It is autosomal recessive

E. Patients with the disease rarely have systemic findings
Menkes Disease in an X-linked recessive disorder caused by mutations of the ATP7A gene encoding
a copper-transporting ATPase. It is lethal in males. Systemic findings include severe psychomotor
retardation, seizures, and growth failure. Diagnosis is made by low serum ceruloplasmin


74) The disorder in the figure is:

A. Cutis verticis gyrata Correct Choice

B. Discoid lupus

C. Tinea capitis

D. Dissecting cellulites

E. Alopecia mucinosis
This is a picture of cutis vertices gyrata, a condition which can be seen in association with primary
amyloidosis


75) Patients with this syndrome are at increased risk for developing Lhermite-Duclos disease:

A. Fabry's Disease

B. MEN Type IIA

C. Bourneville's Disease

D. Nail-Patella Syndrome

E. Cowden's SyndromeCorrect Choice
Patients with Cowden's Syndrome (multiple hamartoma syndrome) are at increased risk for
Lhermite Duclos disease (dysplastic gangliocytoma of the cerebellum).


76) The disorder shown in the figure is:

A. Pretibial myxedema

B. Lichen amyloidosis

C. Lichen planus Correct Choice

D. Lichen simplex chronicus

E. Psoriasis
These lesions are characteristic for lichen planus.Lichen planus is a rare disorder in young children
causing violaceous papules with sharp demarcation and scales. You can induce more of these
lesions by scratching the skin surface (Koebner phenomenon). The most common form of the
disorder is the acute eruptive form where lesions erupt like a viral exanthem and cover most of the




                                                  22
body surface. Treatment is supportive—antihistamines, topical steroids and rarely systemic steroid
therapy


77) The peak sensitivity to prophyrins occurs at which wavelengths?

A. 320-400nm

B. 290-320nm

C. 400-410nmCorrect Choice

D. 410-450nm

E. 220-290nm
The Soret band (400-410 nm) is the portion of ultraviolet wavelengths at which most porphyrins are
most sensitive


78) The patient demonstrated in figure 6 would have:

A. LDL receptor deficiency

B. Decreased triglycerides

C. Decreased LDL

D. Decreased chylomicrons

E. Elevated triglycerides Correct Choice
This patient has eruptive xanthomas, a condition which is classically associated with elevated serum
triglycerides


79) One might see all of the following laboratory and clinical abnormalities in cryoglobulinemia
associated with hepatitis C virus infection EXCEPT:


A. Acrocyanosis

B. Urticarial plaques

C. Elevated liver function tests

D. Elevated C3 levelsCorrect Choice

E. Positive rheumatoid factor
Laboratory abnormalities of HCV include an elevation of liver enzymes, positive rheumatoid factor
(70-90%), and DEPRESSED C3 levels. Classical clinical presentation includes palpable purpura,
arthralgias, and glomerulonephritis. Livedo reticularis, hemorrhagic bullae, acrocyanosis, and
urticarial plaques can also be found


80) Carotenemia can be a manifestation of:

A. Graves disease

B. Amyloidosis

C. Porphyria

D. Pretibial myxedema

E. HypothyroidismCorrect Choice




                                                 23
Reduced matabolism of beta-carotene in the diet, can result in yellowing of the skin in
hypothyroidism. Treatment of porphyria with beta-carotene can result in carotenemia. Pretibial
myxedema, Graves disease and amyloidosis do not result in carotenemia


81) A patient presents with mild mental retardation, infertility, joint contractures, short stature,
ichthyosis, and sparse hair with trichoschisis. All of the following are true regarding this patient's
condition EXCEPT:


A. The syndrome is caused by impaired nucleotide excision repair

B. Patients may have associated cataracts

C. Perifoveal glistening white dots are a featureCorrect Choice

D. If photosensitivity is a feature, gonad size may be normal

E. This syndrome is inherited in an autosomal recessive manner
The patient described has IBIDS syndrome (Ichthyosis, Brittle hair, Intellectual impairment,
Decreased fertility, and Short stature), or Tay's syndrome. This syndrome is autosomal recessive,
caused by mutations in the ERCC2/XPD or ERCC3/XPB genes, resulting in impaired nucleotide
excision repair. Cataracts may be a feature. If photosensitivity is a feature (PIBIDS), gonal size is
normal. Perifoveal glistening white dots are a feature of Sjorgen-Larsson Syndrome, not IBIDS


82) This syndrome is cause by defects in the genes that code for tumor suppressor proteins
hamartin and tuberin.


A. Bourneville's DiseaseCorrect Choice

B. Cronkhite-Canada Syndrome

C. Blue Rubber Bleb Nevus Syndrome

D. Bannayan-Riley-Rubalcaba Syndrome

E. Cowden's Syndrome
Tuberous sclerosis, also known as Bourneville's Disease, is an autosomal dominant neurocutaneous
disorder due to mutations in the TSC1 and TSC2 genes which code for the tumor suppressor
proteins hamartin and tuberin, respectively


83) Which of the following is characteristic of diabetic skin?

A. Candida tropicalis is the most common cause of angular cheilitis

B. The level of cleavage in bullous diabeticorum is subcorneal

C. Yellow skin may occur in up to 10% of diabeticsCorrect Choice

D. There is a well-established association between deep granuloma annulare (GA) and diabetes

E. Approximately 20% of diabetics have necrobiosis lipoidica diabeticorum (NLD)
Perhaps 0.3 to 3% of diabetics have NLD, whereas approximately 20% of NLD patients have
diabetes or glucose intolerance. The split in bullous diabeticorum is normally either intraepidermal
or subepidermal. C. albicans is the most common cause of diabetes-related yeast infections. The
association between GA and diabetes is controversial, but if the two are related, generalized and
perforating GA have been implicated. Yellow skin may occur in up to 10% of diabetic patients and is
characterized by diffuse yellow-orange skin. Half of these patients have elevated serum carotene
levels. The suggested is cause is elevated consumption of yellow fruits and vegetables in the setting
of impaired hepatic metabolism of carotene and subsequent non-enzymatic glycosylation of dermal
collagen




                                                   24
84) All of the following are true regarding Henoch-Schonlein Purpura EXCEPT:

A. It may be complicated by intussusception

B. It is self-resolving

C. Direct immunofluorescence of lesional and peri-lesional skin will demonstrate C3 and fibrin
deposits in small vessel wall

D. It is an IgG mediated small vessel vasculitisCorrect Choice

E. It is often preceded by an upper respiratory infection
Henoch-Schonlein Purpura (HSP) is an IgA mediated small vessel vasculitis. Clinically, patients
present with palpable purpura of the lower extremities and buttocks, GI vasculitis, and
glomerulonephritis


85) Which of the following statements about necrolytic migratory erythema is TRUE?

A. Serum glucagon levels are usually normal

B. The offending tumor originates from endocrine argentaffin cells

C. Vacuolar changes are normally found on routine histology

D. Acanthosis and parakeratosis are found on routine histologyCorrect Choice

E. There is a low incidence of metastasis of the offending tumor at the time of diagnosis
Necrolytic migratory erythema or glucagonoma syndrome is clinically characterized by periorificial
and acral erythema, vesicles, pustules and erosions. A circinate pattern is often seen. Glossitis and
cheilitis are features as well. The underlying tumor is an alpha-2 glucagon producing islet cell
pancreatic carcinoma, which is metastatic at the time of diagnosis in the majority of cases.
Histopathological findings include dyskeratotic keratinocytes in the stratum granulosum, acanthosis
and parakeratosis. Serum glucagon levels are elevated. The tumors must be resected if feasible,
and intravenous somatostatin and amino acids have been used for treatment


86) The deficient enzyme of the condition seen in the figure is:

A. Uroporphyrinogen decarboxylase Correct Choice

B. Ferrochetalase

C. dALA synthetase

D. Uroporphyrinogen synthetase

E. Porphobilinogen deaminase
A patient with porphyria cutanea tarda is shown. The associated enzyme deficiency is
uroporphyrinogen decarboxylase


87) Which of the following is NOT true regarding calciphylaxis?

A. May be present with retiform purpura

B. Patients with proximally-located lesions have a better prognosis than those with acral
lesionsCorrect Choice

C. Has an associated mortality of 60-80%

D. May be treated with parathyroidectomy

E. Histologic findings include medial calcification and intimal hyperplasia of small arteries and



                                                   25
arterioles
Distribution of lesions in calciphylaxis may predict prognosis; those with acral lesions have a better
outcome than those with proximally located lesions


88) Porphyria cutanea tarda may be associated with all of the following except:

A. Estrogens

B. Hepatitis C virus infection

C. Alcohol

D. Inherited deficiency of uroporphyrinogen III synthase Correct Choice

E. Polyhalogenated hydrocarbons
A homozygous defect in uroporphyrinogen III synthase in a cause of congenital erythropoietic
porphyria, not porphyria cutanea tarda (PCT). PCT may be caused by sporadic or familial deficiency
in uroporphyrinogen decarboxylase, or by any of the other causes listed above




                                                  26

Mais conteúdo relacionado

Mais procurados

ETAS_MCQ_03 a genodermatoses
ETAS_MCQ_03 a genodermatosesETAS_MCQ_03 a genodermatoses
ETAS_MCQ_03 a genodermatosesDerma202
 
ETAS_MCQ_03 b genodermatoses
ETAS_MCQ_03 b genodermatosesETAS_MCQ_03 b genodermatoses
ETAS_MCQ_03 b genodermatosesDerma202
 
ETAS_MCQ_14 plants and creatures of dermatologic significance
ETAS_MCQ_14 plants and creatures of dermatologic significanceETAS_MCQ_14 plants and creatures of dermatologic significance
ETAS_MCQ_14 plants and creatures of dermatologic significanceDerma202
 
ETAS_MCQ_01 structures of skin
ETAS_MCQ_01 structures of skinETAS_MCQ_01 structures of skin
ETAS_MCQ_01 structures of skinDerma202
 
Mock question paper for neet pg, usmle, plab and fmge (mci screening exam) on...
Mock question paper for neet pg, usmle, plab and fmge (mci screening exam) on...Mock question paper for neet pg, usmle, plab and fmge (mci screening exam) on...
Mock question paper for neet pg, usmle, plab and fmge (mci screening exam) on...Medico Apps
 
ETAS_MCQ_02 immunodermatology
ETAS_MCQ_02 immunodermatologyETAS_MCQ_02 immunodermatology
ETAS_MCQ_02 immunodermatologyDerma202
 
Previous year question on pemphigus vulgaris based on neet pg, usmle, plab an...
Previous year question on pemphigus vulgaris based on neet pg, usmle, plab an...Previous year question on pemphigus vulgaris based on neet pg, usmle, plab an...
Previous year question on pemphigus vulgaris based on neet pg, usmle, plab an...Medico Apps
 
Internal Medicine Image Challenge MCQs
Internal Medicine Image Challenge MCQsInternal Medicine Image Challenge MCQs
Internal Medicine Image Challenge MCQsSherif Elbadrawy
 
Previous year question on lichen planus based on neet pg, usmle, plab and fmg...
Previous year question on lichen planus based on neet pg, usmle, plab and fmg...Previous year question on lichen planus based on neet pg, usmle, plab and fmg...
Previous year question on lichen planus based on neet pg, usmle, plab and fmg...Abhishek Gupta
 
Previous year question on tonsils based on neet pg, usmle, plab and fmge or m...
Previous year question on tonsils based on neet pg, usmle, plab and fmge or m...Previous year question on tonsils based on neet pg, usmle, plab and fmge or m...
Previous year question on tonsils based on neet pg, usmle, plab and fmge or m...Abhishek Gupta
 
Previous year question on cataract based on neet pg, usmle, plab and fmge or ...
Previous year question on cataract based on neet pg, usmle, plab and fmge or ...Previous year question on cataract based on neet pg, usmle, plab and fmge or ...
Previous year question on cataract based on neet pg, usmle, plab and fmge or ...Medico Apps
 
Previous year question on leptospirosis based on neet pg, usmle, plab and fmg...
Previous year question on leptospirosis based on neet pg, usmle, plab and fmg...Previous year question on leptospirosis based on neet pg, usmle, plab and fmg...
Previous year question on leptospirosis based on neet pg, usmle, plab and fmg...Medico Apps
 
Previous year question on polio based on neet pg, usmle, plab and fmge or mci...
Previous year question on polio based on neet pg, usmle, plab and fmge or mci...Previous year question on polio based on neet pg, usmle, plab and fmge or mci...
Previous year question on polio based on neet pg, usmle, plab and fmge or mci...Abhishek Gupta
 
Vesiculo bullous II
Vesiculo bullous IIVesiculo bullous II
Vesiculo bullous IIIAU Dent
 
Vitiligo in association with Erythema dyschromicum perstans
Vitiligo in association with Erythema dyschromicum perstansVitiligo in association with Erythema dyschromicum perstans
Vitiligo in association with Erythema dyschromicum perstansVR Foundation
 
Previous year question on staining based on neet pg, usmle, plab and fmge or ...
Previous year question on staining based on neet pg, usmle, plab and fmge or ...Previous year question on staining based on neet pg, usmle, plab and fmge or ...
Previous year question on staining based on neet pg, usmle, plab and fmge or ...Abhishek Gupta
 
Vohwinkel Syndrome vs. KID Syndrome
Vohwinkel Syndrome vs. KID Syndrome Vohwinkel Syndrome vs. KID Syndrome
Vohwinkel Syndrome vs. KID Syndrome JenniferArmstrong6
 

Mais procurados (20)

ETAS_MCQ_03 a genodermatoses
ETAS_MCQ_03 a genodermatosesETAS_MCQ_03 a genodermatoses
ETAS_MCQ_03 a genodermatoses
 
ETAS_MCQ_03 b genodermatoses
ETAS_MCQ_03 b genodermatosesETAS_MCQ_03 b genodermatoses
ETAS_MCQ_03 b genodermatoses
 
Derma.
Derma.Derma.
Derma.
 
ETAS_MCQ_14 plants and creatures of dermatologic significance
ETAS_MCQ_14 plants and creatures of dermatologic significanceETAS_MCQ_14 plants and creatures of dermatologic significance
ETAS_MCQ_14 plants and creatures of dermatologic significance
 
ETAS_MCQ_01 structures of skin
ETAS_MCQ_01 structures of skinETAS_MCQ_01 structures of skin
ETAS_MCQ_01 structures of skin
 
Mock question paper for neet pg, usmle, plab and fmge (mci screening exam) on...
Mock question paper for neet pg, usmle, plab and fmge (mci screening exam) on...Mock question paper for neet pg, usmle, plab and fmge (mci screening exam) on...
Mock question paper for neet pg, usmle, plab and fmge (mci screening exam) on...
 
ETAS_MCQ_02 immunodermatology
ETAS_MCQ_02 immunodermatologyETAS_MCQ_02 immunodermatology
ETAS_MCQ_02 immunodermatology
 
Mrcp Part 2 Witten Exam
Mrcp Part 2 Witten ExamMrcp Part 2 Witten Exam
Mrcp Part 2 Witten Exam
 
Previous year question on pemphigus vulgaris based on neet pg, usmle, plab an...
Previous year question on pemphigus vulgaris based on neet pg, usmle, plab an...Previous year question on pemphigus vulgaris based on neet pg, usmle, plab an...
Previous year question on pemphigus vulgaris based on neet pg, usmle, plab an...
 
Internal Medicine Image Challenge MCQs
Internal Medicine Image Challenge MCQsInternal Medicine Image Challenge MCQs
Internal Medicine Image Challenge MCQs
 
Previous year question on lichen planus based on neet pg, usmle, plab and fmg...
Previous year question on lichen planus based on neet pg, usmle, plab and fmg...Previous year question on lichen planus based on neet pg, usmle, plab and fmg...
Previous year question on lichen planus based on neet pg, usmle, plab and fmg...
 
MRCP MOCK EXAM
MRCP MOCK EXAMMRCP MOCK EXAM
MRCP MOCK EXAM
 
Previous year question on tonsils based on neet pg, usmle, plab and fmge or m...
Previous year question on tonsils based on neet pg, usmle, plab and fmge or m...Previous year question on tonsils based on neet pg, usmle, plab and fmge or m...
Previous year question on tonsils based on neet pg, usmle, plab and fmge or m...
 
Previous year question on cataract based on neet pg, usmle, plab and fmge or ...
Previous year question on cataract based on neet pg, usmle, plab and fmge or ...Previous year question on cataract based on neet pg, usmle, plab and fmge or ...
Previous year question on cataract based on neet pg, usmle, plab and fmge or ...
 
Previous year question on leptospirosis based on neet pg, usmle, plab and fmg...
Previous year question on leptospirosis based on neet pg, usmle, plab and fmg...Previous year question on leptospirosis based on neet pg, usmle, plab and fmg...
Previous year question on leptospirosis based on neet pg, usmle, plab and fmg...
 
Previous year question on polio based on neet pg, usmle, plab and fmge or mci...
Previous year question on polio based on neet pg, usmle, plab and fmge or mci...Previous year question on polio based on neet pg, usmle, plab and fmge or mci...
Previous year question on polio based on neet pg, usmle, plab and fmge or mci...
 
Vesiculo bullous II
Vesiculo bullous IIVesiculo bullous II
Vesiculo bullous II
 
Vitiligo in association with Erythema dyschromicum perstans
Vitiligo in association with Erythema dyschromicum perstansVitiligo in association with Erythema dyschromicum perstans
Vitiligo in association with Erythema dyschromicum perstans
 
Previous year question on staining based on neet pg, usmle, plab and fmge or ...
Previous year question on staining based on neet pg, usmle, plab and fmge or ...Previous year question on staining based on neet pg, usmle, plab and fmge or ...
Previous year question on staining based on neet pg, usmle, plab and fmge or ...
 
Vohwinkel Syndrome vs. KID Syndrome
Vohwinkel Syndrome vs. KID Syndrome Vohwinkel Syndrome vs. KID Syndrome
Vohwinkel Syndrome vs. KID Syndrome
 

Semelhante a Manifestations of systemic diseases

100 mc qsintropicalmedicine
100 mc qsintropicalmedicine100 mc qsintropicalmedicine
100 mc qsintropicalmedicineElyas Mohammed
 
Syndromes associate with periodontium
Syndromes associate with periodontiumSyndromes associate with periodontium
Syndromes associate with periodontiumVaibhaviNandgaonkar
 
Cutaneous presentation of tumours
Cutaneous presentation of tumoursCutaneous presentation of tumours
Cutaneous presentation of tumoursmeducationdotnet
 
Bmj33300181
Bmj33300181Bmj33300181
Bmj33300181lalumee
 
1) MEASLES, RUBELLA, VARICELLA & HERPES ZOOSTER.ppt
1) MEASLES, RUBELLA, VARICELLA & HERPES ZOOSTER.ppt1) MEASLES, RUBELLA, VARICELLA & HERPES ZOOSTER.ppt
1) MEASLES, RUBELLA, VARICELLA & HERPES ZOOSTER.pptBibout
 
Deramatology MRCGP Qs
Deramatology MRCGP QsDeramatology MRCGP Qs
Deramatology MRCGP Qsssnsharifa
 
Cutaneous manifestations of internal malignancy and paraneoplastic syndromes
Cutaneous manifestations of internal malignancy and paraneoplastic syndromes Cutaneous manifestations of internal malignancy and paraneoplastic syndromes
Cutaneous manifestations of internal malignancy and paraneoplastic syndromes gamal sultan
 
Epidermodysplasia verruciformis and HPV in Immunocompromised
Epidermodysplasia verruciformis and HPV in ImmunocompromisedEpidermodysplasia verruciformis and HPV in Immunocompromised
Epidermodysplasia verruciformis and HPV in ImmunocompromisedDrYusraShabbir
 
Infections and salivary gland disease in pediatric age: how to manage - Slide...
Infections and salivary gland disease in pediatric age: how to manage - Slide...Infections and salivary gland disease in pediatric age: how to manage - Slide...
Infections and salivary gland disease in pediatric age: how to manage - Slide...WAidid
 
NEPHROTIC SYNDROME IN PAEDIATRIC
NEPHROTIC SYNDROME IN PAEDIATRICNEPHROTIC SYNDROME IN PAEDIATRIC
NEPHROTIC SYNDROME IN PAEDIATRICMona Mofti
 
Dermatology board review
Dermatology board reviewDermatology board review
Dermatology board reviewAhmed Amer
 
Premalignantlesions and conditions by Dr. Amit T. Suryawanshi, Oral Surgeon,...
Premalignantlesions and conditions by Dr. Amit T. Suryawanshi,  Oral Surgeon,...Premalignantlesions and conditions by Dr. Amit T. Suryawanshi,  Oral Surgeon,...
Premalignantlesions and conditions by Dr. Amit T. Suryawanshi, Oral Surgeon,...All Good Things
 
Premalignantlesions and conditions by Dr. Amit Suryawanshi .Oral & Maxillofac...
Premalignantlesions and conditions by Dr. Amit Suryawanshi .Oral & Maxillofac...Premalignantlesions and conditions by Dr. Amit Suryawanshi .Oral & Maxillofac...
Premalignantlesions and conditions by Dr. Amit Suryawanshi .Oral & Maxillofac...All Good Things
 
Medical student surgery osce
Medical student surgery osceMedical student surgery osce
Medical student surgery osceBashir BnYunus
 
Esophageal cancer
Esophageal cancer Esophageal cancer
Esophageal cancer Ahmad shu
 
Webinar on mucormycosis
Webinar on mucormycosisWebinar on mucormycosis
Webinar on mucormycosisAnjanaMohite
 
Cervical mases DDx and Radio-imaging by B.H.A.A Malik
Cervical mases DDx and Radio-imaging by  B.H.A.A MalikCervical mases DDx and Radio-imaging by  B.H.A.A Malik
Cervical mases DDx and Radio-imaging by B.H.A.A Malikbushra a malik
 
Kurdistan Board GEH J Club git-Skin CONNECTIONS.
Kurdistan Board GEH J Club  git-Skin CONNECTIONS.Kurdistan Board GEH J Club  git-Skin CONNECTIONS.
Kurdistan Board GEH J Club git-Skin CONNECTIONS.Shaikhani.
 

Semelhante a Manifestations of systemic diseases (20)

100 mc qsintropicalmedicine
100 mc qsintropicalmedicine100 mc qsintropicalmedicine
100 mc qsintropicalmedicine
 
Syndromes associate with periodontium
Syndromes associate with periodontiumSyndromes associate with periodontium
Syndromes associate with periodontium
 
Cutaneous presentation of tumours
Cutaneous presentation of tumoursCutaneous presentation of tumours
Cutaneous presentation of tumours
 
Bmj33300181
Bmj33300181Bmj33300181
Bmj33300181
 
1) MEASLES, RUBELLA, VARICELLA & HERPES ZOOSTER.ppt
1) MEASLES, RUBELLA, VARICELLA & HERPES ZOOSTER.ppt1) MEASLES, RUBELLA, VARICELLA & HERPES ZOOSTER.ppt
1) MEASLES, RUBELLA, VARICELLA & HERPES ZOOSTER.ppt
 
Deramatology MRCGP Qs
Deramatology MRCGP QsDeramatology MRCGP Qs
Deramatology MRCGP Qs
 
Mrcp 2 dermatology
Mrcp 2 dermatologyMrcp 2 dermatology
Mrcp 2 dermatology
 
multiple myloma.pptx
multiple myloma.pptxmultiple myloma.pptx
multiple myloma.pptx
 
Cutaneous manifestations of internal malignancy and paraneoplastic syndromes
Cutaneous manifestations of internal malignancy and paraneoplastic syndromes Cutaneous manifestations of internal malignancy and paraneoplastic syndromes
Cutaneous manifestations of internal malignancy and paraneoplastic syndromes
 
Epidermodysplasia verruciformis and HPV in Immunocompromised
Epidermodysplasia verruciformis and HPV in ImmunocompromisedEpidermodysplasia verruciformis and HPV in Immunocompromised
Epidermodysplasia verruciformis and HPV in Immunocompromised
 
Infections and salivary gland disease in pediatric age: how to manage - Slide...
Infections and salivary gland disease in pediatric age: how to manage - Slide...Infections and salivary gland disease in pediatric age: how to manage - Slide...
Infections and salivary gland disease in pediatric age: how to manage - Slide...
 
NEPHROTIC SYNDROME IN PAEDIATRIC
NEPHROTIC SYNDROME IN PAEDIATRICNEPHROTIC SYNDROME IN PAEDIATRIC
NEPHROTIC SYNDROME IN PAEDIATRIC
 
Dermatology board review
Dermatology board reviewDermatology board review
Dermatology board review
 
Premalignantlesions and conditions by Dr. Amit T. Suryawanshi, Oral Surgeon,...
Premalignantlesions and conditions by Dr. Amit T. Suryawanshi,  Oral Surgeon,...Premalignantlesions and conditions by Dr. Amit T. Suryawanshi,  Oral Surgeon,...
Premalignantlesions and conditions by Dr. Amit T. Suryawanshi, Oral Surgeon,...
 
Premalignantlesions and conditions by Dr. Amit Suryawanshi .Oral & Maxillofac...
Premalignantlesions and conditions by Dr. Amit Suryawanshi .Oral & Maxillofac...Premalignantlesions and conditions by Dr. Amit Suryawanshi .Oral & Maxillofac...
Premalignantlesions and conditions by Dr. Amit Suryawanshi .Oral & Maxillofac...
 
Medical student surgery osce
Medical student surgery osceMedical student surgery osce
Medical student surgery osce
 
Esophageal cancer
Esophageal cancer Esophageal cancer
Esophageal cancer
 
Webinar on mucormycosis
Webinar on mucormycosisWebinar on mucormycosis
Webinar on mucormycosis
 
Cervical mases DDx and Radio-imaging by B.H.A.A Malik
Cervical mases DDx and Radio-imaging by  B.H.A.A MalikCervical mases DDx and Radio-imaging by  B.H.A.A Malik
Cervical mases DDx and Radio-imaging by B.H.A.A Malik
 
Kurdistan Board GEH J Club git-Skin CONNECTIONS.
Kurdistan Board GEH J Club  git-Skin CONNECTIONS.Kurdistan Board GEH J Club  git-Skin CONNECTIONS.
Kurdistan Board GEH J Club git-Skin CONNECTIONS.
 

Mais de Derma202

Phototherapy treatment protocol
Phototherapy treatment protocolPhototherapy treatment protocol
Phototherapy treatment protocolDerma202
 
Histopathplogical photos
Histopathplogical photosHistopathplogical photos
Histopathplogical photosDerma202
 
Slide study from ETAS
Slide  study from ETASSlide  study from ETAS
Slide study from ETASDerma202
 
Arab board primary exam in dermatology 2012
Arab board primary exam  in dermatology 2012Arab board primary exam  in dermatology 2012
Arab board primary exam in dermatology 2012Derma202
 
Dermatology
DermatologyDermatology
DermatologyDerma202
 
Derm handbook for medical students and junior doctors 2010
Derm handbook for medical students and junior doctors 2010Derm handbook for medical students and junior doctors 2010
Derm handbook for medical students and junior doctors 2010Derma202
 

Mais de Derma202 (6)

Phototherapy treatment protocol
Phototherapy treatment protocolPhototherapy treatment protocol
Phototherapy treatment protocol
 
Histopathplogical photos
Histopathplogical photosHistopathplogical photos
Histopathplogical photos
 
Slide study from ETAS
Slide  study from ETASSlide  study from ETAS
Slide study from ETAS
 
Arab board primary exam in dermatology 2012
Arab board primary exam  in dermatology 2012Arab board primary exam  in dermatology 2012
Arab board primary exam in dermatology 2012
 
Dermatology
DermatologyDermatology
Dermatology
 
Derm handbook for medical students and junior doctors 2010
Derm handbook for medical students and junior doctors 2010Derm handbook for medical students and junior doctors 2010
Derm handbook for medical students and junior doctors 2010
 

Último

SWD (Short wave diathermy)- Physiotherapy.ppt
SWD (Short wave diathermy)- Physiotherapy.pptSWD (Short wave diathermy)- Physiotherapy.ppt
SWD (Short wave diathermy)- Physiotherapy.pptMumux Mirani
 
Pharmaceutical Marketting: Unit-5, Pricing
Pharmaceutical Marketting: Unit-5, PricingPharmaceutical Marketting: Unit-5, Pricing
Pharmaceutical Marketting: Unit-5, PricingArunagarwal328757
 
Measurement of Radiation and Dosimetric Procedure.pptx
Measurement of Radiation and Dosimetric Procedure.pptxMeasurement of Radiation and Dosimetric Procedure.pptx
Measurement of Radiation and Dosimetric Procedure.pptxDr. Dheeraj Kumar
 
Presentation on General Anesthetics pdf.
Presentation on General Anesthetics pdf.Presentation on General Anesthetics pdf.
Presentation on General Anesthetics pdf.Prerana Jadhav
 
call girls in aerocity DELHI 🔝 >༒9540349809 🔝 genuine Escort Service 🔝✔️✔️
call girls in aerocity DELHI 🔝 >༒9540349809 🔝 genuine Escort Service 🔝✔️✔️call girls in aerocity DELHI 🔝 >༒9540349809 🔝 genuine Escort Service 🔝✔️✔️
call girls in aerocity DELHI 🔝 >༒9540349809 🔝 genuine Escort Service 🔝✔️✔️saminamagar
 
PULMONARY EDEMA AND ITS MANAGEMENT.pdf
PULMONARY EDEMA AND  ITS  MANAGEMENT.pdfPULMONARY EDEMA AND  ITS  MANAGEMENT.pdf
PULMONARY EDEMA AND ITS MANAGEMENT.pdfDolisha Warbi
 
PULMONARY EMBOLISM AND ITS MANAGEMENTS.pdf
PULMONARY EMBOLISM AND ITS MANAGEMENTS.pdfPULMONARY EMBOLISM AND ITS MANAGEMENTS.pdf
PULMONARY EMBOLISM AND ITS MANAGEMENTS.pdfDolisha Warbi
 
VarSeq 2.6.0: Advancing Pharmacogenomics and Genomic Analysis
VarSeq 2.6.0: Advancing Pharmacogenomics and Genomic AnalysisVarSeq 2.6.0: Advancing Pharmacogenomics and Genomic Analysis
VarSeq 2.6.0: Advancing Pharmacogenomics and Genomic AnalysisGolden Helix
 
POST NATAL EXERCISES AND ITS IMPACT.pptx
POST NATAL EXERCISES AND ITS IMPACT.pptxPOST NATAL EXERCISES AND ITS IMPACT.pptx
POST NATAL EXERCISES AND ITS IMPACT.pptxvirengeeta
 
Statistical modeling in pharmaceutical research and development.
Statistical modeling in pharmaceutical research and development.Statistical modeling in pharmaceutical research and development.
Statistical modeling in pharmaceutical research and development.ANJALI
 
Glomerular Filtration rate and its determinants.pptx
Glomerular Filtration rate and its determinants.pptxGlomerular Filtration rate and its determinants.pptx
Glomerular Filtration rate and its determinants.pptxDr.Nusrat Tariq
 
Let's Talk About It: To Disclose or Not to Disclose?
Let's Talk About It: To Disclose or Not to Disclose?Let's Talk About It: To Disclose or Not to Disclose?
Let's Talk About It: To Disclose or Not to Disclose?bkling
 
COVID-19 (NOVEL CORONA VIRUS DISEASE PANDEMIC ).pptx
COVID-19  (NOVEL CORONA  VIRUS DISEASE PANDEMIC ).pptxCOVID-19  (NOVEL CORONA  VIRUS DISEASE PANDEMIC ).pptx
COVID-19 (NOVEL CORONA VIRUS DISEASE PANDEMIC ).pptxBibekananda shah
 
call girls in Connaught Place DELHI 🔝 >༒9540349809 🔝 genuine Escort Service ...
call girls in Connaught Place  DELHI 🔝 >༒9540349809 🔝 genuine Escort Service ...call girls in Connaught Place  DELHI 🔝 >༒9540349809 🔝 genuine Escort Service ...
call girls in Connaught Place DELHI 🔝 >༒9540349809 🔝 genuine Escort Service ...saminamagar
 
PNEUMOTHORAX AND ITS MANAGEMENTS.pdf
PNEUMOTHORAX   AND  ITS  MANAGEMENTS.pdfPNEUMOTHORAX   AND  ITS  MANAGEMENTS.pdf
PNEUMOTHORAX AND ITS MANAGEMENTS.pdfDolisha Warbi
 
Radiation Dosimetry Parameters and Isodose Curves.pptx
Radiation Dosimetry Parameters and Isodose Curves.pptxRadiation Dosimetry Parameters and Isodose Curves.pptx
Radiation Dosimetry Parameters and Isodose Curves.pptxDr. Dheeraj Kumar
 
Informed Consent Empowering Healthcare Decision-Making.pptx
Informed Consent Empowering Healthcare Decision-Making.pptxInformed Consent Empowering Healthcare Decision-Making.pptx
Informed Consent Empowering Healthcare Decision-Making.pptxSasikiranMarri
 
Hematology and Immunology - Leukocytes Functions
Hematology and Immunology - Leukocytes FunctionsHematology and Immunology - Leukocytes Functions
Hematology and Immunology - Leukocytes FunctionsMedicoseAcademics
 
epilepsy and status epilepticus for undergraduate.pptx
epilepsy and status epilepticus  for undergraduate.pptxepilepsy and status epilepticus  for undergraduate.pptx
epilepsy and status epilepticus for undergraduate.pptxMohamed Rizk Khodair
 

Último (20)

Epilepsy
EpilepsyEpilepsy
Epilepsy
 
SWD (Short wave diathermy)- Physiotherapy.ppt
SWD (Short wave diathermy)- Physiotherapy.pptSWD (Short wave diathermy)- Physiotherapy.ppt
SWD (Short wave diathermy)- Physiotherapy.ppt
 
Pharmaceutical Marketting: Unit-5, Pricing
Pharmaceutical Marketting: Unit-5, PricingPharmaceutical Marketting: Unit-5, Pricing
Pharmaceutical Marketting: Unit-5, Pricing
 
Measurement of Radiation and Dosimetric Procedure.pptx
Measurement of Radiation and Dosimetric Procedure.pptxMeasurement of Radiation and Dosimetric Procedure.pptx
Measurement of Radiation and Dosimetric Procedure.pptx
 
Presentation on General Anesthetics pdf.
Presentation on General Anesthetics pdf.Presentation on General Anesthetics pdf.
Presentation on General Anesthetics pdf.
 
call girls in aerocity DELHI 🔝 >༒9540349809 🔝 genuine Escort Service 🔝✔️✔️
call girls in aerocity DELHI 🔝 >༒9540349809 🔝 genuine Escort Service 🔝✔️✔️call girls in aerocity DELHI 🔝 >༒9540349809 🔝 genuine Escort Service 🔝✔️✔️
call girls in aerocity DELHI 🔝 >༒9540349809 🔝 genuine Escort Service 🔝✔️✔️
 
PULMONARY EDEMA AND ITS MANAGEMENT.pdf
PULMONARY EDEMA AND  ITS  MANAGEMENT.pdfPULMONARY EDEMA AND  ITS  MANAGEMENT.pdf
PULMONARY EDEMA AND ITS MANAGEMENT.pdf
 
PULMONARY EMBOLISM AND ITS MANAGEMENTS.pdf
PULMONARY EMBOLISM AND ITS MANAGEMENTS.pdfPULMONARY EMBOLISM AND ITS MANAGEMENTS.pdf
PULMONARY EMBOLISM AND ITS MANAGEMENTS.pdf
 
VarSeq 2.6.0: Advancing Pharmacogenomics and Genomic Analysis
VarSeq 2.6.0: Advancing Pharmacogenomics and Genomic AnalysisVarSeq 2.6.0: Advancing Pharmacogenomics and Genomic Analysis
VarSeq 2.6.0: Advancing Pharmacogenomics and Genomic Analysis
 
POST NATAL EXERCISES AND ITS IMPACT.pptx
POST NATAL EXERCISES AND ITS IMPACT.pptxPOST NATAL EXERCISES AND ITS IMPACT.pptx
POST NATAL EXERCISES AND ITS IMPACT.pptx
 
Statistical modeling in pharmaceutical research and development.
Statistical modeling in pharmaceutical research and development.Statistical modeling in pharmaceutical research and development.
Statistical modeling in pharmaceutical research and development.
 
Glomerular Filtration rate and its determinants.pptx
Glomerular Filtration rate and its determinants.pptxGlomerular Filtration rate and its determinants.pptx
Glomerular Filtration rate and its determinants.pptx
 
Let's Talk About It: To Disclose or Not to Disclose?
Let's Talk About It: To Disclose or Not to Disclose?Let's Talk About It: To Disclose or Not to Disclose?
Let's Talk About It: To Disclose or Not to Disclose?
 
COVID-19 (NOVEL CORONA VIRUS DISEASE PANDEMIC ).pptx
COVID-19  (NOVEL CORONA  VIRUS DISEASE PANDEMIC ).pptxCOVID-19  (NOVEL CORONA  VIRUS DISEASE PANDEMIC ).pptx
COVID-19 (NOVEL CORONA VIRUS DISEASE PANDEMIC ).pptx
 
call girls in Connaught Place DELHI 🔝 >༒9540349809 🔝 genuine Escort Service ...
call girls in Connaught Place  DELHI 🔝 >༒9540349809 🔝 genuine Escort Service ...call girls in Connaught Place  DELHI 🔝 >༒9540349809 🔝 genuine Escort Service ...
call girls in Connaught Place DELHI 🔝 >༒9540349809 🔝 genuine Escort Service ...
 
PNEUMOTHORAX AND ITS MANAGEMENTS.pdf
PNEUMOTHORAX   AND  ITS  MANAGEMENTS.pdfPNEUMOTHORAX   AND  ITS  MANAGEMENTS.pdf
PNEUMOTHORAX AND ITS MANAGEMENTS.pdf
 
Radiation Dosimetry Parameters and Isodose Curves.pptx
Radiation Dosimetry Parameters and Isodose Curves.pptxRadiation Dosimetry Parameters and Isodose Curves.pptx
Radiation Dosimetry Parameters and Isodose Curves.pptx
 
Informed Consent Empowering Healthcare Decision-Making.pptx
Informed Consent Empowering Healthcare Decision-Making.pptxInformed Consent Empowering Healthcare Decision-Making.pptx
Informed Consent Empowering Healthcare Decision-Making.pptx
 
Hematology and Immunology - Leukocytes Functions
Hematology and Immunology - Leukocytes FunctionsHematology and Immunology - Leukocytes Functions
Hematology and Immunology - Leukocytes Functions
 
epilepsy and status epilepticus for undergraduate.pptx
epilepsy and status epilepticus  for undergraduate.pptxepilepsy and status epilepticus  for undergraduate.pptx
epilepsy and status epilepticus for undergraduate.pptx
 

Manifestations of systemic diseases

  • 1. Manifestations of systemic diseases 1) Bony abnormalities seen in patients with the condition in the figure include all of the following except: A. spina bifida B. pseudoarthrosis C. lordosis D. monoarticular arthropathy Correct Choice E. Kyphosis This picture demonstrates a café au lait macule and kyphoscoliosis in a patient with Neurofibromatosis type 1 (NF1). The bony abnormalities seen in these patients include: kyphoscoliosis, pseudoarthrosis, lordosis, and spina bifida 2) The organism that causes this infection shown in the figure is: A. C. albicans Correct Choice B. P. aeruginoas C. T. tonsuran D. T. verrucosum E. C. immitis The organism that causes erosio interdigitalis blastomycetica is candida albicans. 3) Erythema gyratum repens is known to be associated with all of the following malignancies except: A. Bladder carcinoma B. Cervical carcinoma C. Breast carcinoma D. Lung carcinoma E. Gastric carcinomaCorrect Choice Erythema gyratum repens presents more commonly in men than in women, and appears clinically as concentric erythematous rings with trailing scale on the trunk and proximal extremities. The skin is described as having a "wood grain" appearance. Skin findings often precede the diagnosis of an associated malignancy. Lung carcinoma is the most commonly associated malignancy, but it has also been reported in association with breast, cervical, bowel, and bladder cancer. 4) Regarding eruptive xanthomas, which of the following is true? A. They occur in the setting of familial hyperlipidemia types I, IV, and VCorrect Choice B. They occur in the setting of familial hyperlipidemia types II and III C. They are most commonly found on the eyelids D. They are not related to alcohol consumption E. Triglyceride levels are usually below 500mg/dl 1
  • 2. Eruptive xanthomas generally occur in patients with triglyceride levels of 2000mg/dl or greater. Associations include poorly-controlled diabetes mellitus, retinoids, estrogens, excessive alcohol consumption (leading to pancreatitis) and familial hyperlipidemias types I, IV and V. Clinically, they appear as crops of firm, non-tender yellowish papules with an erythematous border. Most commonly, they occur on the extensor surfaces, but they can be diffuse. A reduction in triglycerides and/or tight glucose control usually results in a reduction in the number of lesions 5) Paraneoplastic pemphigus is associated with all of the following underlying malignancies EXCEPT: A. Non-Hodgkins lymphoma B. Castleman's tumor C. Chronic lymphocytic leukemia D. Thymoma E. Lung carcinomaCorrect Choice Paraneoplastic pemphigus has been associated with Non-Hodgkins lymphoma, chronic lymphocytic leukemia, thymoma, Castleman's tumor, and sarcoma. It has not been associated with lung carcinoma. Treatment includes management of the underlying malignancy, as well as prednisone or other immunosuppressive agents 6) What is the most common malignancy associated with this condition at this location? A. Gastric cancerCorrect Choice B. Melanoma C. Breast cancer D. Thyroid cancer E. Pancreatic cancer Malignant acanthosis nigricans usually presents with sudden onset and is rapidly progressive. It may be associated with diffuse keratodermas of the palms and soles or eruptive seborrheic keratoses 7) The most common location of the lesions in nephrogenic fibrosing dermopathy is: A. Chest B. Palms and soles C. Face D. Back E. Lower extremities Correct Choice The most common location for lesions of nephrogenic fibrosis dermopathy is the lower extremities 8) A 55 year old patient presents with new onset brown macules on arms, legs, face and palms. She gives a 3 month history of diarrhea, abdominal cramps, weight loss and protein-losing enteropathy. The most likely diagnosis is: A. Ulcerative colitis B. Cowden disease C. Cronkhite-Canada syndromeCorrect Choice 2
  • 3. D. Plummer-Vinson syndrome E. Peutz-Jeghers syndrome Cronkhite-Canada is an aquired disease characterized by the development of polyps throughout the GI tract. Patients can present with hyperpigmented macules as well as the sequella of GI malabsorption. The malignant transformation of polyps can occur 9) Which of the following is not a feature of Cronkhite-Canada syndrome? A. Hyperpigmented macules B. Diarrhea C. Alopecia D. Lung carcinomaCorrect Choice E. Dystrophic nails Cronkhite-Canada syndrome is a rare, non-familial disease characterized by patchy alopecia, nail changes or loss, lentigines, inflammatory polyps, abdominal pain, and a protein losing enteropathy 10) A patient with congenital hypertrophy of retinal epithelium is most likely to have: A. Adenomatous polyposisCorrect Choice B. Tram-track calcifications on head radiograph C. Peg-shaped teeth D. Pheochromocytoma E. An autosomal dominant mutation in the MSH2 gene Congenital hyperpigmentation of the retinal pigment (CHRPE) is an early feature of Gardner syndrome (GS). It is found in approximately 60% of patients with GS. GS is an autosomal dominant disorder characterized by precancerous intestinal polyposis and subsequent adenocarcinoma of the gastrointestinal tract. Cutaneous manifestations include epidermoid cysts, osteomas, desmoids and fibrous tumors. A mutation in the adenomatous polyposis coli (APC) gene, a tumor suppressor gene, is responsible for the disease. Most patients develop colon carcinoma by the 2nd or 3rd decade. Therefore, prophylactic colectomy is warranted. Mutations in the MSH2 gene are found in Muir-Torre syndrome. Pheochromocytomas are found in multiple endocrine neoplasia (MEN) syndromes IIa and IIb. Tram track calcifications are found in Sturge-Weber syndrome. Peg-shaped teeth are found in multiple syndromes including ectodermal dysplasia 11) Which of the following may be associated with Graves’ disease? A. Geographic tongue B. Madarosis C. Dermatitis herpetiformisCorrect Choice D. Hypohidrosis E. Thick, pale lips Graves’ disease is a thyrotoxic condition that results from the production of thyroid-stimulating immunoglobulins (TSI) by stimulated B lymphocytes. The TSI bind to the thyroid-stimulating hormone (TSH) receptor and mimic TSH thereby stimulating thyroid growth and thyroid hormone overproduction. Signs and symptoms of Graves’ disease include goiter, tachycardia, exophthalmos, tremor, sweating, palpitations, smooth moist skin, diarrhea, sleeplessness, irritability, and weight loss. Autoimmune cutaneous disease may also be associated with Graves’ disease including vitiligo, 3
  • 4. dermatitis herpetiformis, herpes gestationis, and pemphigus vulgaris. Cutaneous manifestations of hypothyroidism include xerosis, hyperhidrosis, yellowish hue, myxedema, and purpura. The hair may be dry, brittle and coarse; alopecia may be diffuse and/or involve the lateral eyebrow (madarosis). 12) Which of the following cutaneous findings are characteristic for Vohwinkles Syndrome? A. Transient erythroderma at birth, palmoplantar keratoderma, follicular hyperkeratosis, scarring alopecia, dystrophic nails B. Oral papillomatosis, palmoplantar keratoses, acral keratoses, lipomas, hemangiomas, scrotal tongue C. Honeycombed diffuse palmoplantar keratoderma, pseudoainhum with autoamputation, star- shaped keratosis over knuckles, nail dystrophy, and alopeciaCorrect Choice D. Rapidly progressive alopecia of all hair-bearing areas, onycholysis, onychoschizia, onychomadesis, hyperpigmented macules on extremities E. Dense depigmented lusterless hair, pili torti, doughy skin, diffuse cutaneous hypopigmentation Vohwinkles Syndrome is an autosomal dominant disorder caused by a mutation of the GJB2 gene which encodes Connexin 26. Clinical features include deafness, as well as the cutaneous findings described in choice A 13) In patients with mixed cryoglobulinema associated with hepatitis C, the most likely laboratory abnormality is: A. Decreased cryoglobulins B. Elevated rheumatoid factor Correct Choice C. + p-ANCA D. + ANA E. Elevated hematocrit In patients with mixed cryoglobulinemia, the most likely laboratory abnormality among the options listed is an elevated rheumatoid factor 14) cryoglobulinemia are true EXCEPT: A. Type I is composed of monoclonal IgG and polyclonal IgMCorrect Choice B. Type II is composed of polyclonal IgG and monoclonal IgM C. 80% of cases of mixed cryoglobulinemia are associated with Hepatitis C infection D. Type III is composed of polyclonal IgG and polyclonal IgM E. None of these answers are correct (all statements are true) Type I is composed of monoclonal immunoglobulins 15) Which of the following statements regarding porphyrias is TRUE? A. Delta aminolevulenic acid is the only oxidized porphyrinCorrect Choice B. Acute intermittent porphyria is the most common form of porphyria C. Plasma fluoresces at 410 nm in patients with variegate porphyria 4
  • 5. D. Griseofulvin is safe for those with variegate porphyria E. Elevated uroporphyrins are found in the red blood cells of hepatoerythropoietic porphyria Acute intermittent porphyria (AIP) is the second most common porphyria and is caused by a deficiency in porphobilinogen (PBG) deaminase, which is located in the cytosol. Patients suffer from colicky pain, paralysis and psychiatric disorders. There are no specific skin manifestations. PBG and aminolevulenic acid (ALA) are elevated in the urine. Attacks are precipitated by medications such as barbiturates, estrogen, griseofulvin, and sulfonamides as well as starvation, fever and infection. Treatment includes glucose loading and hematin infusion. Congenital erythropoietic porphyria (CEP) or Günter’s disease is caused by a defect in uroporphyrinogen III synthase, which is found in the cytosol. Patients are extremely photosensitive and erythema, blistering and scarring result. Patients present with red urine early in life along with hypertrichosis and red-stained teeth that fluoresce. Uroporphyrins (URO) are much high than coproporphyrins (COPRO) in the urine. URO is found in the red blood cells (rbcs) and COPRO is found in the stool. The rbcs display stable fluorescence. Porphyria cutanea tarda (PCT) is the most common porphyria and is caused by a deficiency (usually sporadic) in uroporphyrinogen decarboxylase, which is found in the cytosol. Patients present with photosensitivity and blistering of sun-exposed areas, especially the dorsal hands. Hypertrichosis and sclerodermoid changes may occur as well. Liver disease (hepatitis C or alcoholic cirrhosis) is often present and hemochromatosis may be associated. Urine may fluoresce pink or coral-red with Wood’s lamp. URO>COPRO in the urine and low levels of COPRO are found in the stool. Treatments include phlebotomy, antimalarials, and therapy for liver disease if appropriate. Hereditary coproporphyria (HCP) is caused by a deficiency in coproporphyrinogen oxidase, which is found in the mitochondria. One-third of patients are photosensitive, and patients suffer gastrointestinal and neurological symptoms similar to AIP. Urine COPRO is elevated only with attacks, and COPRO is present in the stool. Variegate porphyria is the result of decreased activity of protoporphyrinogen oxidase, which is present in the mitochondria. It combines the skin lesions of PCT with the systemic manifestations of AIP. Urine COPRO:URO is 1:1 or COPRO> URO to distinguish it from PCT, and PROTO is found in the stool. The plasma fluoresces at 626nm. Precipitators and treatments are similar to AIP. Erythropoietic protoporphyria (EPP) is caused by ferrochetalase deficiency, which is present in the mitochondria. Patients experience immediate burning of the skin with sun exposure. Protoporphyrin IX, the only oxidized porphyrin in the heme pathway and absorbs in the Soret band (400-410nm). Patients have erythematous plaques in a photo-distribution. Urine porphyrins are normal. PROTO is found in the rbcs and the stool. Excessive porphyrins deposited in the liver lead to gallstones and cirrhosis. Beta carotene may helpful. Hepatoerythropoietic porphyria (HEP) is essentially a homozygous form of PCT, with deficiency in uroporphyrinogen decarboxylase. It is clinically similar to CEP with red urine and hypertrichosis, vesicles and scarring of sun-exposed skin. URO is present in the urine and COPRO in the stool. PROTO is present in rbcs which distinguishes it from CEP, which was URO in rbcs 16) A young African American patient presents with anemia and spontaneously appearing leg ulcers over both lateral and medial malleoli. The most likely diagnosis is: A. Trauma B. Atherosclerotic disease C. Sickle cell anemiaCorrect Choice D. Factitial dermatitis E. Lupus erythematosus Sickle cell anemia should be considered most likely in any young African American with spontaneous leg ulcers. The ulcers are more common in people with severe anemia. 5
  • 6. 17) The expected histology of a biopsy take from the lesion shown in the image would A. Schumann bodies B. Collagen degeneration Correct Choice C. Granulomatous infiltrate localized to the papillary dermis D. Increased mucin E. Elastic fiber degeneration The histologic findings of necrobiosis lipodica diabeticorum are: dermal granulomatous inflammation in a horizontal pattern, collagen degeneration, and normal or atrophic epidermis 18) Which of the following is NOT true regarding Cushing's Syndrome? A. Corticotropin is suppressed with the administration of dexamethasoneCorrect Choice B. It may be associated with an underlying oat cell lung carcinoma C. Urine cortisol levels are elevated D. It may be associated with hypertension and hypokalemia E. May present with hyperpigmentation and facial plethora In Cushing's Syndrome, corticotropin is NOT suppressed with the administration of dexamethasone. The other statements are true 19) Hepatitis C infection is associated with: A. Gianotti-Crosti syndrome B. Kapsi’s sarcoma C. Mixed cryoglobulinemiaCorrect Choice D. Oral hairy leukoplakia E. Pityriasis rosea Cutaneous manifestation associated with hepatitis C include necrolytic acral erythema, porphyria cutanea tarda, lichen planus, polyarteritis nodosa and mixed cryoglobulinemia. Mixed cryoglobulinemias is a systemic vasculitis with variable manifestations including palpable purpura, arthralgias and weakness 20) Which of the following is true regarding piebaldism? A. It is caused by a mutation in the GJB2 gene B. It is caused by a mutation in the proto-oncogene c-KITCorrect Choice C. It is caused by a defect in a protein subunit of a kinase that activates NFkappaB D. It is caused by defective metabolism of phytanic acid E. It is caused by a deficiency of fatty aldehyde dehydrogenase Piebaldism is caused by an autosomal dominant mutation of the proto-oncogene c-KIT which encodes tyrosine a tyrosine kinase receptor on melanocytes, preventing activation by steel factor 6
  • 7. 21) An end stage AIDS patient with tuberulosis presents with diffuse hyperpigmentation of both sun-exposed and unexposed areas. The palmer creases are markedly hyperpigmented and the patient is very ill. The most likely diagnosis is: A. Kaposi's sarcoma B. Argyria C. Lymphoma D. Addison diseaseCorrect Choice E. Tinea versicolor Addison disease is caused by destruction of the adrenal glands by any cause. Tuberculosis used to be the primary cause, now the most common cause is auto-immune destruction. Other causes of adrenal gland destruction include; coccidiomycosis, cryptococcosis, histoplasmosis, sarcoidosis, metastatic tumor and amyloidosis. 22) A complication seen in women of childbearing age with the condition in the figure is: A. Pulmonary emboli B. Lisch nodules C. Gastrointestinal bleeding D. Pulmonary lymphangioleiomyoma Correct Choice E. Atrial septal defect This picture demonstrates a shagreen patch in a patient with Tuberous sclerosis. A complication occurring in female patients of child-bearing age is pulmonary lymphangioleiomyoma 23) The histopathologic findings shown in the image is seen in patients with: A. Cowden's Syndrome Correct Choice B. Proteus syndrome C. Bannayan-Riley-Ruvalcaba D. Rombo syndrome E. Birt-Hogg-Dube Syndrome This slide shows lobular formations extending from the epidermis with palisading keratinocytes at the periphery of the lobules. These features are consistent with trichelemmoma, a characteristic skin finding of Cowden's disease 24) A representative papule on the dorsal distal finger with a coral bead appearance was biopsied. The pathology of the lesion demonstrated oncocytic histiocytes with a ground glass appearance, the diagnosis is: A. Rheumatoid nodules B. Granuloma annulare C. Multicentric reticulohistiocytosis Correct Choice D. Tendinous xanthomas E. Erythema elevatum diutinum 7
  • 8. This picture and the histologic description are consistent with Multicentric reticulohistiocytosis. Multicentric reticulohistiocytosis is associated with a symmetric polyarthritis as well as malignancy. Reported malignancies include melanoma, sarcoma, leukemia, lymphoma, and carcinomas of the breast, colon, bronchus, cervix, stomach, and ovaries 25) Which of the following is NOT associated with hepatitis C disease? A. Mixed cryoglobulinemia B. Single-stranded DNA viridaeCorrect Choice C. Leukocytoclastic vasculitis D. Uroporphyrinogen decarboxylase deficiency E. Single-stranded RNA viridae Hepatitis C virus (HCV) is a single-stranded RNA virus that is a member of the flaviviridae family. Approximately 20-30% of patients develop symptoms with acute infection and 70% will progress to chronic disease. Porphyria cutanea tarda or PCT (uroporphyrinogen decarboxylase deficiency) is associated with HCV in a substantial percentage of patients; in one study, antibodies to HCV were found in 82% of PCT patients. Up to 80% of mixed cryoglobulinemia (MC) cases are associated with HCV. The incidence of lichen planus in HCV patients varies from region to region (0.1-35%). The histopathology of MC lesions is leukocytoclastic vasculitis. Another relatively common association is polyarteritis nodosa (PAN) which is also related to hepatitis B infection. Finally, generalized pruritis is a common complaint of HCV patients 26) All of the following are true regarding incontinentia pigmenti EXCEPT: A. It is an X-linked recessive disorderCorrect Choice B. Inflammation and blistering may be followed by hyperkeratotic, verrucous lesions C. It is caused by a mutation in the NEMO gene D. It is associated with cerebellar ataxia E. It is associated with coloboma and retinal detachment Incontinentia pigmenti is an X-linked dominant disorder caused by a mutation in the NEMO gene. Females only present at birth with linear lesions of inflammation and blistering (stage 1), followed by hyperkeratotic verrucous areas (stage 2), hyperpigmentation (stage 3), and then atrophy (stage 4). Systemic findings include psychomotor retardation, microcephaly, seizures, cerebellar ataxia, coloboma, and retinal detachment 27) Which of the following is a paraneoplastic disease most often associated with lung carcinoma? A. Acanthosis nigricans B. Dermatomyositis C. Paraneoplastic pemphigus D. Erythroderma E. Hypertrichosis lanuginosa acquisitaCorrect Choice Hypertrichosis lanuginosa acquisita is the abrupt onset of downy, soft, non-pigmented hair of the face, trunk, and extremities. It may have an associated glossitis. It is associated with underlying lung carcinoma and may resolve with treatment of the underlying malignancy 28) Patients with this syndrome, as well as their family members, should undergo periodic surveillance for renal cell carcinoma: 8
  • 9. A. Pseudoxanthoma Elasticum B. Fabry's Disease C. Birt-Hogg-Dube SyndromeCorrect Choice D. Osler-Weber-Rendu E. Nail-Patella Syndrome Strong evidence suggests that BHDS and bilateral renal tumors co-segregate in an autosomal dominant fashion. Histologically, these renal tumors are oncocytomas, chromophobe adenomas, and papillary renal cell carcinoma. Periodic surveillance of patients and their relatives for renal cell carcinoma is recommended 29) You are called to evaluate a patient on the oncology service with the lesions pictured. Which of the following statements is NOT true regarding this disease process? (Photo courtesy of Derm101.com) A. This patient will most likely die from complications of the underlying malignancy. B. Skin biopsy would reveal suprabasal acantholysis and dyskeratosis. C. This patient most likely has an underlying gastric carcinoma.Correct Choice D. Direct immunofluorescence of perilesional skin would demonstrate intercellular IgG and granular C3 at the dermoepidermal junction. E. Skin biopsy would reveal vacuolar interface dermatitis with a lichenoid infiltrate. Paraneoplastic pemphigus presents with painful oral erosions/ stomatitis as well as a polymorphous skin eruption. Most patients die from complications of the underlying malignancy. The condition is associated with Non-Hodgkins lymphoma, chronic lymphocytic leukemia, thymoma, Castleman's tumor, and sarcoma. Statements A, C, D, and E are correct 30) All of the following are true regarding the condition pictured EXCEPT: A. It is associated with HLA-DQ2, HLA-DR3, and HLA-B8 B. Only 20% of patients have a gluten-sensitive enteropathyCorrect Choice C. Perilesional direct immunofluroescence shows granular IgA in the dermal papillae and at the dermoepidermal junction D. Cutaneous findings are due to autoantibodies to epidermal transglutaminase E. It is associated with Hashimoto's thyroiditis All of the statements regarding dermatitis herpetiforms, or "Duhring's Disease," are true except for statement B. Virtually all DH patients have gluten-sensitive enteropathy, although only 20% of them have symptoms (such as diarrhea, steatorrhea, weight loss, bloating, and malabsorption). 31) Which of the following is true regarding cutaneous associations with hepatitis C virus (HCV) infection? A. Pruritus in the setting of chronic HCV infection is generally correlated with elevated bile salt levels in the setting of liver failure B. Cutaneous reactions to HCV treatment are less common with interferon/ribavirin combination treatment than with treatment with interferon alone C. HCV-related porphyria cutanea tarda may be caused by decompartmentalization of iron stores and resultant oxidation of uroporphyrinogen decarboxylaseCorrect Choice 9
  • 10. D. Polyarteritis nodosa is associated with HCV infection but not hepatitis B virus infection E. Erosive mucosal lichen planus has a weaker association with HCV than does cutaneous lichen planus Erosive mucosal variant of lichen planus has the strongest association with HCV. The pathogenesis of pruritus in the setting of chronic HCV infection may be related to elevated bile salt levels in the setting of liver failure, but there is not always a direct correlation between serum bile salt level and degree of pruritus. Polyarteritis nodosa is associated with both HCV infection and hepatitis B virus infection. Statement D is correct. Cutaneous reactions to HCV treatment are more common with interferon/ribavirin combination treatment than with treatment with interferon alone. 32) Which of the following is TRUE about diabetic-related skin disease? A. Necrobiosis lipoidica affects approximately 20% of diabetics B. Bullae are common on the thighs C. Diabetic dermopathy affects the upper back D. Yellow skin affects the majority of diabetic patients E. Direct immunofluorenscence is usually negative in bullous diabeticorumCorrect Choice Approximately 30% of diabetic patients have cutaneous manifestations of their disease. Diabetic dermopathy, or “shin spots” is the most common cutaneous association with diabetes. Patients are generally long-standing diabetics, and are associated with symmetrical brownish, atrophic plaques on the shins. Acanthosis nigricans is more common in black and Hispanic diabetics, and may be a component of the HAIRAN (hyperandrogen, insulin resistance, acanthosis nigricans) syndrome. Hyperpigmented velvety plaques are most often found in the flexures. Waxy, thick skin and limited joint mobility are both related to poor glucose control. Scleredema diabeticorum, which most often causes thickened plaques of the upper back and neck most often affects type II diabetics. Necrobiosis lipoidica diabeticorum (NLD) is present in only 0.3 to 3% of diabetics, but perhaps 20% of patients with NLD have diabetes or glucose intolerance. Sharply-demarcated yellow-brown, telangiectatic plaques are found on the tibial surfaces. Ulceration may occur. Yellow skin may affect approximately 10% of diabetics and presents diffusely. Serum carotene levels may be elevated. Perforating disorders may be related, especially in the setting of renal disease. In the setting of hypertriglyceridemia, eruptive xanthomata may be present. The association between diabetes and granuloma annulare (GA) is controversial. If truly an association, the generalized and perforating forms of GA are most commonly associated. Infectious associations include candidiasis, which may present an angular cheilitis, chronic paronychia, interdigital or intertriginous disease. Cutaneous bacterial infections, including group A and B streptococci, pseudomonas aerugenosa and corynebacterium are more common. Dermatophyte infections are not more common, although tinea pedis may increase the risk for cellulitis in diabetic patients. Finally, rhinocerebral mucormycosis is a potential disasterous complication of uncontrolled hyperglycemia with ketoacidosis. Mortality rates can approach 35%. 33) The most likely underlying malignancy associated with the lesions shown is: A. Breast Cancer B. Waldenstrom's macroglobulinemia C. Acute myelogenous leukemia Correct Choice D. Castleman's tumor E. Multiple myeloma These are typical lesions of Acute Febrile Neutrophilic dermatosis (Sweet's Syndrome), a paraneoplastic disease most strongly associated with Acute myelogenous leukemia 34) The most common autoimmune disease associated with hepatitis C is: 10
  • 11. A. Myasthenia gravis B. Autoimmune thyroiditis Correct Choice C. Aplastic anemia D. Rheumatoid arthritis E. Lymphocytic sialadenitis The most common autoimmune disease seen in association with Hepatitis C is autoimmune thyroiditis. 35) This patient with the above findings also has recurrent bouts of rectal bleeding and cramping. The most likely underlying disease is: A. Henoch-schonlein purpura B. Gardner’s syndrome C. Polyarteritis nodosum D. Ulcerative colitis Correct Choice E. Dermatitis herpetiformis These cobblestone mucosal lesions are pyostomatitis vegetans. They are seen in those with ulcerative colitis 36) The shoulder pad sign has been described in which disease? A. cushings disease B. nephrogenic fibosing dermopathy C. dermatomyositis D. systemic lupus erythematosis E. systemic amyloidosisCorrect Choice The shoulder pad sign has been described in the setting of late systemic amyloidosis and is due to the direct deposition of amyloid in the deltoid muscles 37) Hypothyroidism may result in all of the following cutaneous findings except: A. Purpura B. Yellowish hue C. All of these answers are correctCorrect Choice D. Increase in the percentage of telogen hairs E. Madarosis All of the above are non-specific cutaneous manifestations of hypothyroidism: an increase in percentage of telogen hairs, yellowish hue caused by carotenemia, purpura secondary to impaired wound healing, and loss of the lateral third of the eyeborw (madarosis) 38) The most likely underlying diagnosis of the patient in the figure is: A. SLE 11
  • 12. B. Endocarditis C. Mycosis Fungoides D. Multicentric Reticulohistiocytosis E. Dermatomyositis Correct Choice This patient has periungal telangiectasias. It is commonly seen in those with dermatomyositis 39) Which of the following diseases is caused by an enzymatic defect that occurs in the mitochondria? A. Erythropoeitic ProtoporphyriaCorrect Choice B. Hepatoerythropoetic Porphyria C. Congenital Erythropoeitic Porphyria D. Acute Intermittent Porphyria E. Porphyria Cutanea Tarda Defects in many of the enzymes involved in heme synthesis are responsible for porphyrias. The first and last three steps of the heme synthesis pathway occur in the mitochondria 40) Which of the following is (are) characteristic of basal cell nevus syndrome? A. Tram-track calcifications B. Telangiectasias C. Autosomal recessive inheritance D. NEMO gene mutation E. ColobomasCorrect Choice Basal cell nevus syndrome (Gorlin syndrome) is caused by an autosomal dominant mutation in PTCH gene that encodes PTC protein involved in sonic hedge hog pathway. This mutation leads to loss of inhibition of smoothened (SMO) leading to an increased expression of other genes. Cutaneous findings include nevoid basal cell carcinomas, milial cysts, epidermoid cysts and palmar- plantar pits. Neurological findings include calcifications of the falx cerebri, agenesis of the corpus callosum, mental retardation, medulloblastoma and spina bifida. Ocular findings include blindness, cataracts, colobomas and strabismus. Other findings include odontogenic cysts of the jaw, frontal bossing, bifid ribs, and pectus deformity. NEMO gene mutations are found in incontinentia pigmenti. Tram-track calfications are seen Sturge-Weber syndrome. Telangiectasias are seen in ataxia- telangiectasia among others 41) All of the following statements are true regarding this condition EXCEPT: A. 0.3-3% of diabetics have this skin condition B. There is no impact of tight glucose control on the likelihood of developing this condition C. This condition is associated with increased dermal mucinCorrect Choice D. Diabetes or glucose intolerance is found in 20% of these patients E. This condition may be associated with cutaneous anesthesia, hypohidrosis, and partial alopecia Necrobiosis Lipoidica Diabeticorum (NLD) is found in 0.3-3% of diabetics. Approximately 20% of NLD patients have diabetes or glucose intolerance. It presents with single or multiple red-brown papules which progress to sharply demarcated yellow-brown atrophic, telangiectatic plaques with violaceous, irregular borders; common sites include the shins. Cutaneous anesthesia, hypohidrosis, 12
  • 13. and partial alopecia can be found. Pathology shows palisading granulomas containing degenerating collagen (necrobiosis); with NO increase in dermal mucin. There is no impact of tight glucose control on the likelihood of developing NLD 42) A patient with esophageal carcinoma has scale plaques on the ears, hands and feet. He may have: A. Dermatomyositis B. Sweet’s syndrome C. Bazex’s syndromeCorrect Choice D. Paraneoplastic pemphigus E. Multicentric reticulohistiocytosis Bazex’s syndrome (acrokeratosis paraneoplatica of Bazex) is characterized by psoriasiform changes of the hands, ears, feet, and nose. Involvement of the nails may be indistinguishable from psoriasis. It occurs in association with malignancies of the upper aerodigestive tract 43) Which of the following is an autosomal recessive disorder characterized by pili torti and deafness? A. Waardenburg Syndrome B. Bjornstad's SyndromeCorrect Choice C. Refsum Disease D. Sjorgen-Larsson Syndrome E. Cockayne Syndrome Waardenburg Syndrome is an autosomal dominant disorder. Sjorgen-Larsson Syndrome is an autosomal recessive disorder characterized by a triad of ichthyosis, di- or tetraplegia, mental retardation. Refsum Disease is an autosomal recessive disorder whose features include deafness, mild ichthyosis in adulthood, and accentuated palmar creases. Bjornstad's Syndrome is an autosomal recessive disorder characterized by pili torti and deafness. Cockayne Syndrome is an autosomal recessive disorder whose features include photosensitivity, bird-headed facies, and "Mickey-Mouse" ears, as well as deafness 44) Triple palms are a cutaneous manifestation associated with which of the following malignancies? A. Renal carcinoma B. Colon carcinoma C. Pancreatic carcinoma D. Prostate carcinoma E. Lung carcinomaCorrect Choice Tripe palms are rugose thickening of the palms which are nearly always associated with internal malignancy. When tripe palms are present without other cutaneous findings, the most likely internal malignancy is lung cancer. Tripe palms in association with acanthosis nigricans is most likely to be associated with gastric cancer 45) Mutations in the STK11 gene encoding a serine threonine kinase are seen in: 13
  • 14. A. Muir-Torre Syndrome B. Bannayan-Riley-Ruvalcaba Syndrome C. Cronkhite-Canada Syndrome D. Peutz-Jeghers SyndromeCorrect Choice E. Birt-Hogg-Dubbe Syndrome Peutz-Jeghers Syndrome is an autosomal dominant syndrome. 50% of patients have mutations in the STK11 gene which encodes a serine threonine kinase. It presents with periorificial and mucosal lentigines beginning in infancy, as well as hamartomatous intestinal polyps with intussusception and bleeding; gallbladder, pancreatic, breast, ovarian, and testicular cancer 46) When metastases to the skin occur from a thyroid malignancy, they are usually due to: A. Medullary carcinoma B. Papillary adenocarcinomaCorrect Choice C. Cutaneous metastases have not been reported in association with thyroid malignancies D. Anaplastic carcinoma E. Follicular carcinoma Metastases to the skin from a thyroid malignancy are rare, but most reported cases occur with papillary adenocarcinoma 47) An uncommon complication of treatment with potassium iodide is: A. Acute generalized exanthematous pustulosis B. Exacerbation of lichen planus C. Erythema nodosum D. Wolff-Chiakoff effect Correct Choice E. Hyperhidrosis An uncommon complication of treatment with potassium iodide is the Wolff-Chiakoff effect 48) A patient presents with diffuse waxy keratoderma of the palms and soles as well as oral hairy leukoplakia. Which of the following statements regarding this condition is NOT true? A. Features may include squamous cell carcinomas arising from keratodermic skin B. None of these answers are correct (all are true) C. Family members should be advised to undergo cancer screening D. This condition is autosomal recessiveCorrect Choice E. This condition may be associated with esophageal carcinoma Howell-Evans syndrome is an autosomal dominant disorder that presents with diffuse waxy keratoderm of the palms and soles, as well as oral hairy leukoplakia and squamous cell carcinoma arising from keratodermic skin. It is associated with esophageal carcinoma; cancer screening in family members is advised 49) The presence of antibodies to c-ANCA is characteristically seen in patients with which disease? 14
  • 15. A. Nodular vasculitis B. Polyarteritis nodosa C. Churg-Strauss disease D. Ulverative colitis E. Wegener’s granulornatosis Correct Choice Wegener’s granulomatosis is a necrotizing granulomatous disorder that most commonly affects the upper and lower respiratory tracts, kidneys, and eye. Cytoplasmic pattern antineutrophil cytoplasmic autoantibody (anti -proteinase-3) is often positive in this disease. Churg-Strauss is associated with a positive p-ANCA 50) Patients with the condition shown who also have JXG are at increased risk for developing: A. Juvenile chronic myelogenous leukemia Correct Choice B. Non-Hodgkin's Lymphoma C. Medullary carcinoma of the thyroid gland D. Breast cancer E. Esophageal cancer The picture demonstrates a plexiform neuroma in a patient with NF I. Patient with NF 1 who have multiple juvenile xanthogranulomas are at 20 times greater risk for developing juvenile chronic myelogenous leukemia 51) Hyperkeratotic follicular nasal papules have been described as a paraneoplastic sign in the setting of which neoplasm? A. Castleman's tumor B. Renal cell carcinoma C. AML D. Adenocarcinoma of the lung E. Multiple myelomaCorrect Choice Hyperkeratotic follicular nasal papules have been described as a paraneoplastic phenomenon in the setting of multiple myeloma. AML is associated with Sweet's syndrome and Castleman's tumor is associated with paraneoplastic pemphigus 52) A patient with gluten-sensitive enteropathy presents with vesicles on the extensor surfaces of the extremities. What findings are most likely on a perilesional biopsy? A. Granular IgA at the dermoepidermal junction on direct immunofluorescenceCorrect Choice B. Linear C3 and IgG at the dermoepidermal junction on direct immunofluorescence C. Leukocytoclastic vasculitis D. Linear IgA surrounding vessels on direct immunofluorescence E. Pautrier’s micro-abscesses in the epidermis on H & E This patient has dermatitis herpetiformis (DH). DH is characterized by itchy papulovesicles on extensor surfaces of the extremities. Neutrophilic infiltrates at the dermal papillae with vesicle formation are found on histopathologic examination. On immunofluorescence, granular IgA (directed against transglutaminase) deposits are found in perilesional skin. Over 90% of patients 15
  • 16. have gluten-sensitive enteropathy of varying severity. Dapsone is almost universally therapeutic for the skin disease, but not the enteropathy. A gluten free diet, although difficult to maintain, treats the enteropathy. The majority of DH patients have the HLA class II DQ2 genotype. On indirect immunofluorescence, one may find antigliadin, antiendomyseal or antireticulin antibodies. Autoimmune thyroid disease (especially Hashimoto’s thyroiditis), enteropathy-associated T-cell non- Hodgkin’s lymphoma and insulin-dependent diabetes are the most common autoimmune associations. Pautrier’s micro-abscesses are found in mycosis fungoides. Leukocytoclastic vasculitis is not a feature of DH 53) Patients with Werner’s syndrome typically experience which of the following types of cardiac disease? A. Premature atherosclerosisCorrect Choice B. Mitral valve prolapse C. Hypertrophic cardiomyopathy D. Aortic aneurysms E. Cardiomegaly Werner’s syndrome or Progeria is caused by autosomal recessive mutations in WRN (Recql2) gene, which encodes DNA helicase. This defect leads to defects in DNA repair and replication. Patients prematurely age and essentially experience many diseases of aging early in childhood and teen years. They exhibit tight atrophic skin, relatively large heads for body size, leg ulcers and cannities. Patients experience early, accelerated atherosclerosis leading to death by myocardial infarction. In addition, type II diabetes, cataracts, osteoarthritis, osteoporosis and hypogonadism are features 54) A patient presents with a rapid increase in the number and size of these lesions. Your next step should be to: A. Reassure the patient that the lesions are benign B. Order a chest xray C. Refer the patient for a workup to rule out a gastrointestinal malignancy.Correct Choice D. Discuss with the patient's PCP re/ altering their diabetic regimen E. Order a CBC and SPEP/UPEP The sign of Lesser-Trelat is a rapid increase in the size and/or number of seborrheic keratoses. It can be seen in association with acanthosis nigricans and tripe palms, and may have an associated generalized pruritus. It is associated with gastric or colon carcinoma, can appear before or after the malignancy, and may improve with treatment of the underlying malignancy 55) A 6 month-old has a verrucous plaque on the mucosal surface of the lower lip. Skin biopsy is consistent with Riga-Fede disease. You should refer the patient to: A. A hematologist B. An otolarngologist C. A gastroenterologist D. An ophthalmologist E. A neurologistCorrect Choice Riga-Fede disease is a benign ulcerative granulomatous process that occurs in reaction to chronic, repetitive trauma of the oral mucosa by the teeth. Clinically, it appears as firm, verrucous plaques. It may be associated with an underlying developmental anomaly or underlying neurologic disorder 16
  • 17. 56) The clinical findings in figure 3 are associated with which disorder? A. Dermatomyositis B. Graves disease Correct Choice C. Rheumatoid Arthritis D. Myasthenia gravis E. Sarcoidosis This slide shows pretibial myxedema in a patient with Grave’s disease 57) All of the following are true regarding Cockayne Syndrome EXCEPT: A. None of these answers are correct Correct Choice B. It is caused by an inability to repair cyclobutane dimers C. Inheritance is autosomal recessive D. It is associated with retinal pigment degeneration with a "salt and pepper" appearance E. It is associated with basal ganglia calcifications Cockayne Syndrome is an autosomal recessive disorder caused by a mutation of an unknown gene that results in the inability to repair cyclobutane dimers induced by UV exposure. Cutaneous findings include photosensitivity, "bird-headed" facies, and "Mickey Mouse" ears. Statements A-D are true regarding the syndrome 58) Squamous cell carcinoma is seen in which syndrome? A. Nicolau-Balus syndrome B. Bazex syndrome Correct Choice C. Gorlin syndrome D. Rombo syndrome E. Rasmussen syndrome Basex syndrome, also paraneoplastic acrokeratosis, presents with symmetric erythematous, nearly violaceous, psoriasiform dermatoses of the hands, feet, ears and nose. The syndrome is nearly associated with an underlying malignancy, usually squamous cell carcinoma of the upper aerodigestive tract 59) Which of the following is true regarding treatment of inflammatory dermatoses with potassium iodide? A. The Wolff-Chaikoff effect must be considered B. The Wolff-Chaikoff effect must be considered and Binding of excess organic iodide in the thryoid gland may occur C. Thyroid hormone synthesis may be inhibited D. All of these answers are correctCorrect Choice E. Binding of excess organic iodide in the thryoid gland may occur 17
  • 18. The Wolff-Chaikoff effect is described as the binding of excess organic iodide in the thyroid gland with resultant inhibition of thyroid hormone synthesis. This can occur in the setting of patients with erythema nodosum (or other inflammatory dermatoses) being treated with potassium iodide 60) Which of the following statements regarding multiple endocrine neoplasia syndromes is true? A. MEN Type IIA is also known as Wermer's Syndrome B. Multiple mucosal neuromas are seen in association with MEN Type IIA C. Lichen or macular amyloidosis is seen in association with MEN Type IIACorrect Choice D. Patients with MEN Type IIB are at increased risk for developing follicular thyroid carcinoma E. MEN Type IIB is also known as Sipple's Syndrome MEN Type I is also known as Wermer's Syndrome. MEN Type IIA is also known as Sipple's Syndrome. Mucosal neuromas are seen in association with MEN Type IIB. Statement D is correct. Patients with MEN Type IIB are at increased risk for developing medullary thyroid carcinoma 61) All of the following are true regarding the condition pictured EXCEPT: A. Men with this condition may be at increased risk for lymphoma B. Associated calcinosis cutis may be seen in pediatric patients C. Age-appropriate cancer screening is recommended D. May be associated with a psoriasiform scalp dermatitis E. Elevated aldolase is more specific than CPKCorrect Choice All of the statements regarding dermatomyositis are true except for statement E. Elevated muscle enzymes are found in dermatomyositis, but elevated CPK is more specific than aldolase. Diagnosis is also established with muscle biopsy showing evidence of inflammation and abnormal EMG 62) This autosomal dominant condition is characterized by trichodiscomas, fibroepithelial polyps, and fibrofolliculomas: A. POEMS syndrome B. Cowden's disease C. Rombo syndrome D. Proteus syndrome E. Birt-Hogg-Dube syndrome Correct Choice Birt-Hogg-Dube is an autosomal dominant condition characterized by trichodiscomas, fibroepilethial polyps and fibrofolliculomas 63) A patient on hemodialysis presents with indurated plaques having a peau d’orange texture on the bilateral lower legs. Which of the following statements is TRUE? A. The face is usually is affected B. Dapsone will likely be effective treatment C. The diagnostic histopathological findings include acanthosis and hyperkeratosis D. The palms and soles are usually affected 18
  • 19. E. Serum protein electrophoresis should be performedCorrect Choice This patient likely has nephrogenic fibrosing dermopathy (NFD). NFD is an acquired, idiopathic disorder that occurs in renal disease patients. It resembles scleroderma or eosinophilic fasciitis clinically and scleromyxedema histopathologically. Large areas of indurated skin with fibrotic nodules and plaques develop. The extremities are most commonly involved, followed by the trunk. The face, palms and soles are almost never involved. Histopathologically, NFD displays a proliferation of dermal fibroblasts and dendritic cells, thickened collagen bundles, increased elastic fibers, and mucin deposition. Serum protein electrophoresis and immunoelectrophoresis results are negative unlike scleromyxedema, and may be helpful to distinguish the two diseases. NFD is usually a chronic, progressive condition, and favorable responses to medical intervention are anecdotal 64) All of the following are true regarding calciphylaxis except: A. Seen in end-stage renal disease B. Vascular mural calcification occurs late in the processCorrect Choice C. Association with an elevated calcium: phosphate product D. Association with hyperparathyroidism E. High morality rate is due to sepsis Calciphylaxis is a rare, life threatening disorder that is associated with end stage renal disease. Clinically, patients develop a livedoid purpura and enlarging, tender, indurated subcutaneous plaque typically on the legs or abdomen. These plaques are subject to ischemia, skin necrosis, and secondary infection, sepsis, and death. Histologic examination of calciphylaxis is characterized by the triad of small vessel mural calcivication, extravascular calcification, and vascular thrombosis. It appears that vascular mural calcification is an early and essential process in the development of calciphylaxis 65) All of the following are seen more commonly in Crohn's disease than in ulcerative colitis EXCEPT: A. Perineal fistulas B. Pyostomatitis vegetansCorrect Choice C. Oral cobblestoning D. Perineal fissures E. Polyarteritis nodosa Pyoderma vegetans consists of vegetating plaques and vesicopustules of intertriginous areas than heal with hyperpigmentation. When the process involves mucosal surfaces it is called pyostomatitis vegetans. These processes are associated with ulcerative colitis, not Crohn's disease 66) Which of the following is FALSE regarding Fabry’s disease? A. It is associated with acral parasthesias B. It is inherited in an X-linked recessive fashion C. The etiology is a defect in alpha-galactosidase A D. It may be associated with renal failure E. It is inherited in an X-linked dominant fashionCorrect Choice Fabry’s disease is an X-linked lysosomal disorder that leads to excessive deposition of neutral glycosphingolipids in the vascular endothelium. The disorder is caused by a deficiency of alpha- galactosidase-A leading to progressive endothelial accumulation of glycosphingolipids. This 19
  • 20. accumulation accounts for the associated clinical abnormalities of skin, eye, kidney, heart, brain, and peripheral nervous system. Acroparesthesias are a frequent presenting symptom of Fabry’s disease. The primary cutaneous manifestation are angiokeratomata (angiokeratoma corporis diffusum), which are most common in a “bathing-trunk” distribution. Lens opacities and retinal and conjunctival vascular malformations may be found in the eyes. Patients may experience cardiac disease, stroke or renal failure 67) Which of the following statements about multiple endocrine neoplasia (MEN) syndromes is FALSE? A. MEN I is associated with angiofibromas and collagenomas B. MEN IIa is related to a defect in RET proto-oncogene C. MEN I, IIa and IIb are all inherited in an autosomal dominant fashion D. MEN IIa is associated with multiple mucosal neuromasCorrect Choice E. MEN IIa and IIb are both associated with medullary thyroid carcinoma Multiple endocrine neoplasia (MEN) syndromes are divided into types I, IIa and IIb. All are autosomal dominantly inherited. Type I is associated with mutations in MENI; type IIa and IIb involve mutations of RET, which encodes a tyrosine kinase receptor. Type I has the following cutaneous features: facial angiofibromas, collagenomas, lipomas, hypopigmented macules and café- au-lait macules. Type IIa displays macular amyloidosis, while IIb features multiple mucosal neuromas. Systemic features of type I include peptic ulcer disease (as part of Zollinger-Ellison syndrome), parathyroid hyperplasia or adenoma, pancreatic tumors and pituitary neoplasia. Type IIa is characterized by Zollinger-Ellison syndrome, medullary thyroid carcinoma, pheochromocytoma, and parathyroid neoplasia. Type IIb displays marfanoid habitus, Hirschsprung disease, medulloblastoma, medullary thyroid carcinoma, pheochromocytoma and ocular neuromas 68) Which of the following is true regarding nephrogenic fibrosing dermopathy? A. Is associated with a dramatic increase in dermal mucin B. May be associated with antiphospholipid antibodiesCorrect Choice C. Has a rapidly progressive but reversible course D. Is associated with a paraproteinemia E. Is associated with peripheral eosinophilia Nephrogenic fibrosing dermopathy has an indolent course and treatment is usually not satisfactory. It is not associated with a paraproteinemia or peripheral eosinophilia. Some patients have been reported to have antiphospholipid antibodies. Histopathology demonstrates a minimal to slight increase in dermal mucin 69) All of the following are seen more commonly in ulcerative colitis than in Crohn's disease except: A. Pyoderma gangrenosum B. Erythema nodosum C. Chronic apthous ulcers D. Pyoderma vegetans E. Polyarteritis nodosaCorrect Choice Polyarteritis nodosa is associated with Crohn's disease and not ulcerative colitis 20
  • 21. 70) A patient with multiple red-brown papules arranged around the nail folds and “operaglass fingers” may have all of the following associated findings except: A. Underlying malignancy B. Fever and weight loss C. Multinucleate giant cell infiltrate D. Progressive cutaneous diseaseCorrect Choice E. Mutilating arthritis Multicentric reticulohistiocytosis is a rare disorder characterized by mutilating arthritis and cutaneous nodules. Involvement may also include the mucosa, synovia, and internal organs. On the skin, patients may have non-tender, reddish-brown nodules on the face, hands (coral bead appearance), ears, forearms, scalp, neck, and eyelids. These nodules have a histiocytic infiltrate with multinucleated giant cells with a ground glass appearance. Multicentric reticulohistiocytosis has been reported in association with carcinoma of the colon, breast bonchus, cervix, ovary, stomach, mesothelioma, and melanoma 71) Each of the following demonstrates a vasculitis except: A. Henoch-Schoenlein purpura B. Granuloma faciale C. Type 1 cryoglobulinemiaCorrect Choice D. Leukocytoclastic vasculitis E. Wegener’s granulomatosis In type I cryoglobulinemia, monoclonal IgG or IgM cryoglobulins are found often in association with lymphoma, leukemia, Waldenstrom’s macroglobulinemia, or multiple myeloma. On histopathology, type I cryoglobulinemia is characterized by the deposition of precipitated amorphous cryoglobulins on the endothelium and throughout the vessel wall. The precipitates stain with PAS stain. An inflammatory infiltrate is typically lacking in contrast to mixed cryoglobulinemia (which shows a leukocytoclastic vascultitis) 72) Regarding carcinoid syndrome, which of the following is TRUE? A. The most common location for the tumor is the rectum B. Somatostatin is a preferred treatmentCorrect Choice C. VMA is elevated in the urine D. Symptoms are caused by metastases to the skin E. The tumors should not be removed Carcinoid syndrome is caused by a tumor originating in the endocrine argentaffin cells. In order of descending frequency, the most common locations are appendix > small bowel > rectum. Patients experience facial flushing that descends anatomically, diarrhea, hyperhidrosis and bronchial reactivity. A pellagra-like eruption may occur as well as telangiectasia and sclerodermoid changes. Patients experience symptoms once the disease metastasizes to the liver or if it did not involve the GI tract initially. Diagnosis can be made finding elevated 5-hydroxyindolacetic acid (5-HIAA) in the urine. Nitrosonaphthol turns urine purple if 5-HIAA levels are significantly elevated, and can be used as a screening test. Bananas, tomatoes, plums, avocadoes and eggplant can increase levels of 5- HIAA. Treatment involves removal of the tumor along with medical therapy if appropriate. Agents used include somatostatin, methylsergide, cyproheptadine, and beta blockers. Vanillylmandelic acid (VMA) is an important urinary metabolic product of epinephrine and norepinephrine and is detected in the diagnosis of pheochromocytoma 21
  • 22. 73) Which of the following statements regarding Menkes Disease is true? A. Diagnosis is made by elevated serum ceruloplasmin B. Males with the disease may have cutaneous hypopigmentation C. It is caused by a mutation of the ATP7A geneCorrect Choice D. It is autosomal recessive E. Patients with the disease rarely have systemic findings Menkes Disease in an X-linked recessive disorder caused by mutations of the ATP7A gene encoding a copper-transporting ATPase. It is lethal in males. Systemic findings include severe psychomotor retardation, seizures, and growth failure. Diagnosis is made by low serum ceruloplasmin 74) The disorder in the figure is: A. Cutis verticis gyrata Correct Choice B. Discoid lupus C. Tinea capitis D. Dissecting cellulites E. Alopecia mucinosis This is a picture of cutis vertices gyrata, a condition which can be seen in association with primary amyloidosis 75) Patients with this syndrome are at increased risk for developing Lhermite-Duclos disease: A. Fabry's Disease B. MEN Type IIA C. Bourneville's Disease D. Nail-Patella Syndrome E. Cowden's SyndromeCorrect Choice Patients with Cowden's Syndrome (multiple hamartoma syndrome) are at increased risk for Lhermite Duclos disease (dysplastic gangliocytoma of the cerebellum). 76) The disorder shown in the figure is: A. Pretibial myxedema B. Lichen amyloidosis C. Lichen planus Correct Choice D. Lichen simplex chronicus E. Psoriasis These lesions are characteristic for lichen planus.Lichen planus is a rare disorder in young children causing violaceous papules with sharp demarcation and scales. You can induce more of these lesions by scratching the skin surface (Koebner phenomenon). The most common form of the disorder is the acute eruptive form where lesions erupt like a viral exanthem and cover most of the 22
  • 23. body surface. Treatment is supportive—antihistamines, topical steroids and rarely systemic steroid therapy 77) The peak sensitivity to prophyrins occurs at which wavelengths? A. 320-400nm B. 290-320nm C. 400-410nmCorrect Choice D. 410-450nm E. 220-290nm The Soret band (400-410 nm) is the portion of ultraviolet wavelengths at which most porphyrins are most sensitive 78) The patient demonstrated in figure 6 would have: A. LDL receptor deficiency B. Decreased triglycerides C. Decreased LDL D. Decreased chylomicrons E. Elevated triglycerides Correct Choice This patient has eruptive xanthomas, a condition which is classically associated with elevated serum triglycerides 79) One might see all of the following laboratory and clinical abnormalities in cryoglobulinemia associated with hepatitis C virus infection EXCEPT: A. Acrocyanosis B. Urticarial plaques C. Elevated liver function tests D. Elevated C3 levelsCorrect Choice E. Positive rheumatoid factor Laboratory abnormalities of HCV include an elevation of liver enzymes, positive rheumatoid factor (70-90%), and DEPRESSED C3 levels. Classical clinical presentation includes palpable purpura, arthralgias, and glomerulonephritis. Livedo reticularis, hemorrhagic bullae, acrocyanosis, and urticarial plaques can also be found 80) Carotenemia can be a manifestation of: A. Graves disease B. Amyloidosis C. Porphyria D. Pretibial myxedema E. HypothyroidismCorrect Choice 23
  • 24. Reduced matabolism of beta-carotene in the diet, can result in yellowing of the skin in hypothyroidism. Treatment of porphyria with beta-carotene can result in carotenemia. Pretibial myxedema, Graves disease and amyloidosis do not result in carotenemia 81) A patient presents with mild mental retardation, infertility, joint contractures, short stature, ichthyosis, and sparse hair with trichoschisis. All of the following are true regarding this patient's condition EXCEPT: A. The syndrome is caused by impaired nucleotide excision repair B. Patients may have associated cataracts C. Perifoveal glistening white dots are a featureCorrect Choice D. If photosensitivity is a feature, gonad size may be normal E. This syndrome is inherited in an autosomal recessive manner The patient described has IBIDS syndrome (Ichthyosis, Brittle hair, Intellectual impairment, Decreased fertility, and Short stature), or Tay's syndrome. This syndrome is autosomal recessive, caused by mutations in the ERCC2/XPD or ERCC3/XPB genes, resulting in impaired nucleotide excision repair. Cataracts may be a feature. If photosensitivity is a feature (PIBIDS), gonal size is normal. Perifoveal glistening white dots are a feature of Sjorgen-Larsson Syndrome, not IBIDS 82) This syndrome is cause by defects in the genes that code for tumor suppressor proteins hamartin and tuberin. A. Bourneville's DiseaseCorrect Choice B. Cronkhite-Canada Syndrome C. Blue Rubber Bleb Nevus Syndrome D. Bannayan-Riley-Rubalcaba Syndrome E. Cowden's Syndrome Tuberous sclerosis, also known as Bourneville's Disease, is an autosomal dominant neurocutaneous disorder due to mutations in the TSC1 and TSC2 genes which code for the tumor suppressor proteins hamartin and tuberin, respectively 83) Which of the following is characteristic of diabetic skin? A. Candida tropicalis is the most common cause of angular cheilitis B. The level of cleavage in bullous diabeticorum is subcorneal C. Yellow skin may occur in up to 10% of diabeticsCorrect Choice D. There is a well-established association between deep granuloma annulare (GA) and diabetes E. Approximately 20% of diabetics have necrobiosis lipoidica diabeticorum (NLD) Perhaps 0.3 to 3% of diabetics have NLD, whereas approximately 20% of NLD patients have diabetes or glucose intolerance. The split in bullous diabeticorum is normally either intraepidermal or subepidermal. C. albicans is the most common cause of diabetes-related yeast infections. The association between GA and diabetes is controversial, but if the two are related, generalized and perforating GA have been implicated. Yellow skin may occur in up to 10% of diabetic patients and is characterized by diffuse yellow-orange skin. Half of these patients have elevated serum carotene levels. The suggested is cause is elevated consumption of yellow fruits and vegetables in the setting of impaired hepatic metabolism of carotene and subsequent non-enzymatic glycosylation of dermal collagen 24
  • 25. 84) All of the following are true regarding Henoch-Schonlein Purpura EXCEPT: A. It may be complicated by intussusception B. It is self-resolving C. Direct immunofluorescence of lesional and peri-lesional skin will demonstrate C3 and fibrin deposits in small vessel wall D. It is an IgG mediated small vessel vasculitisCorrect Choice E. It is often preceded by an upper respiratory infection Henoch-Schonlein Purpura (HSP) is an IgA mediated small vessel vasculitis. Clinically, patients present with palpable purpura of the lower extremities and buttocks, GI vasculitis, and glomerulonephritis 85) Which of the following statements about necrolytic migratory erythema is TRUE? A. Serum glucagon levels are usually normal B. The offending tumor originates from endocrine argentaffin cells C. Vacuolar changes are normally found on routine histology D. Acanthosis and parakeratosis are found on routine histologyCorrect Choice E. There is a low incidence of metastasis of the offending tumor at the time of diagnosis Necrolytic migratory erythema or glucagonoma syndrome is clinically characterized by periorificial and acral erythema, vesicles, pustules and erosions. A circinate pattern is often seen. Glossitis and cheilitis are features as well. The underlying tumor is an alpha-2 glucagon producing islet cell pancreatic carcinoma, which is metastatic at the time of diagnosis in the majority of cases. Histopathological findings include dyskeratotic keratinocytes in the stratum granulosum, acanthosis and parakeratosis. Serum glucagon levels are elevated. The tumors must be resected if feasible, and intravenous somatostatin and amino acids have been used for treatment 86) The deficient enzyme of the condition seen in the figure is: A. Uroporphyrinogen decarboxylase Correct Choice B. Ferrochetalase C. dALA synthetase D. Uroporphyrinogen synthetase E. Porphobilinogen deaminase A patient with porphyria cutanea tarda is shown. The associated enzyme deficiency is uroporphyrinogen decarboxylase 87) Which of the following is NOT true regarding calciphylaxis? A. May be present with retiform purpura B. Patients with proximally-located lesions have a better prognosis than those with acral lesionsCorrect Choice C. Has an associated mortality of 60-80% D. May be treated with parathyroidectomy E. Histologic findings include medial calcification and intimal hyperplasia of small arteries and 25
  • 26. arterioles Distribution of lesions in calciphylaxis may predict prognosis; those with acral lesions have a better outcome than those with proximally located lesions 88) Porphyria cutanea tarda may be associated with all of the following except: A. Estrogens B. Hepatitis C virus infection C. Alcohol D. Inherited deficiency of uroporphyrinogen III synthase Correct Choice E. Polyhalogenated hydrocarbons A homozygous defect in uroporphyrinogen III synthase in a cause of congenital erythropoietic porphyria, not porphyria cutanea tarda (PCT). PCT may be caused by sporadic or familial deficiency in uroporphyrinogen decarboxylase, or by any of the other causes listed above 26